<<

VISIT THE AANEM MARKETPLACE AT WWW.AANEM.ORG FOR NEW PRODUCTS AMERICAN ASSOCIATION OF NEUROMUSCULAR & ELECTRODIAGNOSTIC MEDICINE Neuromuscular Update I Toxin Induced Neuromuscular Diseases, Neuromuscular Junction Disorders, and Neuromuscular Vignettes Neuromuscular Update II ICU Related Neuromuscular Complications, Alternative and Rehab Approaches to Managing NMD, and Neuromuscular Vignettes

Photo by Michael D. Stubblefi eld, MD ICU Related Neuromuscular Complications, Alternative and Rehab Approaches to Managing NMD, and Neuromuscular Vignettes (Neuromuscular Update II) Shawn J. Bird, MD Bassam A. Bassam, MD Sunil K. Aggarwal, MD, PhD Gregory T. Carter, MD, MS David G. Polin, MD Bakri Elsheikh, MBBS Zachary Simmons, MD

AANEM 58th Annual Meeting San Francisco, California

Copyright © September 2011 American Association of Neuromuscular & Electrodiagnostic Medicine 2621 Superior Drive NW Rochester, MN 55901

Printed by Johnson’s Printing Company, Inc. 1 Please be aware that some of the medical devices or pharmaceuticals discussed in this handout may not be cleared by the FDA or cleared by the FDA for the specific use described by the authors and are “off-label” (i.e., a use not described on the product’s label). “Off-label” devices or pharmaceuticals may be used if, in the judgment of the treating physician, such use is medically indicated to treat a patient’s condition. Information regarding the FDA clearance status of a particular device or pharmaceutical may be obtained by reading the product’s package labeling, by contacting a sales representative or legal counsel of the manufacturer of the device or pharmaceutical, or by contacting the FDA at 1-800-638-2041.

2 ICU Related Neuromuscular Complications, Alternative and Rehab Approaches to Managing NMD, and Neuromuscular Vignettes (Neuromuscular Update II)

Table of Contents

Course Objectives & Course Committee 4

Faculty 5

Neuromuscular Complications of Critical Illness 7 Shawn J. Bird, MD, and Bassam A. Bassam, MD

Alternative and Rehabilitative Approaches to Managing 17 Sunil K. Aggarwal, MD, PhD and Gregory T. Carter, MD, MS

Neuromuscular Vignettes 27 David G. Polin, MD ,Bakri Elsheikh, MBBS, and Zachary Simmons, MD

CME Questions 33

Dr. Quan held Pfizer stock and is a consultant for and does clinical trials for Genzyme. All conflicts of interest have been resolved according to ACCME standards. All other authors/faculty have nothing to disclose.

Chair: Dianna Quan, MD

The ideas and opinions expressed in this publication are solely those of the specific authors and do not necessarily represent those of the AANEM.

3 Objectives

Objectives - This two-part NM update course (labeled CC and CF) will present participants with clinical cases in NMDs. This course is an excellent review of NM medicine. Update II covers ICU related NMcomplications, alternative and rehabilitative approaches to managing NMD, and a vignette session that will include both neuropathic and myopathic disorders. Participants will acquire skills to (1) utilize a pattern recognition approach elucidated through clinical vignettes in the diagnosis and management of patients with ICU related NM complications, (2) discuss alternative and rehabilitative approaches to managing NMdisorders (3) practice the vignette-based format used for many questions on the NMmedicine board examination. Target Audience: • Neurologists, physical medicine and rehabilitation and other physicians interested in neuromuscular and electrodiagnostic medicine • Health care professionals involved in the management of patients with neuromuscular diseases • Researchers who are actively involved in the neuromuscular and/or electrodiagnostic research Accreditation Statement - The AANEM is accredited by the Accreditation Council for Continuing Medical Education to provide continuing medical education (CME) for physicians. CME Credit - The AANEM designates this live activity for a maximum of 3.25 AMA PRA Category 1 CreditsTM. If purchased, the AANEM designates this enduring material for a maximum of 4.75 AMA PRA Category 1 CreditsTM. This educational event is approved as an Accredited Group Learning Activity under Section 1 of the Framework of Continuing Professional Development (CPD) options for the Maintenance of Certification Program of the Royal College of Physicians and Surgeons of Canada. Physicians should claim only the credit commensurate with the extent of their participation in the activity. CME for this course is available 09/2011 - 09/2014. CEUs Credit - The AANEM has designated this live activity for a maximum of 3.25 AANEM CEUs. If purchased, the AANEM designates this enduring material for a maximum of 4.75 CEUs.

2010-2011 Course Committee

Shawn J. Bird, MD, Chair Taylor B. Harrison, MD A. Arturo Leis, MD Philadelphia, PA Atlanta, GA Jackson, MS

Gary L. Branch, DO Laurence J. Kinsella, MD Marcy C. Schlinger, DO Owosso, MI Saint Louis, MO Okemos, MI

Lawrence W. Frank, MD Shashi B. Kumar, MD Benjamin S. Warfel, MD Elmhurst, IL Tacoma, WA Lancaster, PA

2010-2011 AANEM President

Timothy R. Dillingham, MD, MS Milwaukee, Wisconsin

4 ICU Related Neuromuscular Complications, Alternative and Rehab Approaches to Managing NMD, and Neuromuscular Vignettes (Neuromuscular Update II)

Faculty Sunil K. Aggarwal, MD, PhD Shawn J. Bird, MD Resident Professor Physical Medicine and Rehabilitation Director of Neurdiagnostic Laboratories Rusk Institute of Rehabilitation Medicine Department of New York University University of Pennsylvania New York, New York Philadelphia, Pennsylvania

Sunil Kumar Aggarwal is a graduate of the University of Dr. Bird is director of the Laboratory and Washington’s National Institutes of Health-supported Medical Clinical Neurophysiology Fellowship Program as well as the Scientist Training Program. He received his medical degree in 2010 Clinic at the University of Pennsylvania. He and his doctorate in medical geography in 2008 and completed is a professor of neurology at Penn. He received undergraduate his internship in preliminary internal medicine at Virginia Mason degrees in electrical engineering and biology from Cornell Medical Center in Seattle, Washington. Currently, he is completing University. He attended medical school at the Johns Hopkins his residency in physical medicine and rehabilitation at New York University School of Medicine. He completed his neurology University’s Rusk Institute of Rehabilitation Medicine. He has residency and fellowship training in neuromuscular disease and authored or coauthored papers on cannabinoid medical science, electrodiagnostic medicine at the University of Pennsylvania. He dosing, and human rights published in peer-reviewed journals. He is a member of the American Association of Neuromuscular & served as served as a expert reviewer for the American Medical Electrodiagnostic Medicine (AANEM), is currently chair of the Association Council on Science and Public Health’s report on AANEM’s Course Committee, and has served on the Journal, medical cannabis science. Dr. Aggarwal recently formed a group Special Interest, and Historical committees. He is a member of the called Health Professionals for Responsible Drug Scheduling. American Academy of Neurology and a fellow of the American Neurological Association. His primary research interests include neuromuscular disorders associated with critical illness, immune- Bassam A. Bassam, MD mediated neuropathies, and myasthenia gravis. Professor of Neurology Director, Neuromuscular Program and EMG Laboratory University of South Alabama Gregory T. Carter, MD, MS Mobile, Alabama Department of Clinical Neurosciences Providence Medical Group Dr. Bassam completed residency training in neurology at Wayne Olympia, Washington State University, Detroit Michigan, and neuromuscular disease fellowship at Wayne State University and Mayo Clinic, Rochester, Dr. Carter is medical director of the Muscular Dystrophy Minnesota. He is board certified by the American Board of Association Regional Neuromuscular Center and the Providence Psychiatry and Neurology (ABPN) and the American Board of Hospice and Palliative Care program in Olympia, Washington. His Electrodiagnostic Medicine (ABEM) and is a certified Diplomate current research focuses on improving rehabilitation management in the subspecialty of neuromuscular medicine by the ABPN. Dr. of patients with neuromuscular disease. He has coauthored over Bassam is a member of the American Academy of Neurology and 160 peer-reviewed publications in these areas and currently serves the American Association of Neuromuscular & Electrodiagnostic as senior associate editor for Muscle & Nerve. Medicine (AANEM). He has served on various AANEM and ABEM committees as a member, committee chairperson, or board examiner. He has authored or coauthored scientific articles and book chapters and lectured widely on neuromuscular diseases He also is a frequent ad hoc reviewser for neuromuscular journals. His academic and clinical interests are focused on neuromuscular diseases and needle electromyography. 5 Bakri Elsheikh, MBBS board certified in physical medicine and rehabilitation and in the Assistant Professor of Neurology subspecialties of neuromuscular medicine and pain medicine. Director, EMG Laboratory From 1991 to 2003 he served on the clinical faculty at Temple The Ohio State University Medical Center University School of Medicine. From 1996 to 2001 he served on Columbus, Ohio the board of directors of the Board of Certification, International for certification of orthotists and prosthetists . From 2001 to 2004 Dr. Bakri Elsheikh is currently assistant professor of neurology at he served as chairman of the Department of physical medicine The Ohio State University, where he serves as the director of the and rehabilitation and vice president of the Medical and Dental EMG laboratory. He received his medical degree in 1991 from Staff of Lancaster General Health. From 2004 to 2007 he served the University of Khartoum, Sudan. He completed his internship as president of the Medical and Dental Staff of Lancaster General and general medicine training in Khartoum Teaching Hospital. Health. In addition to electrodiagnostic medicine, Dr. Polin’s He obtained a postgraduate internal medicine degree form the interests include the management of pain and spasticity utilizing Royal College of Physicians in the United Kingdom (MRCP). intrathecal medication. Dr. Polin is board certified by the American He did neurology training and worked as a consultant neurologist Board of Electrodiagnostic Medicine. in Saudi Arabia. He also completed a neurology residency and neurophysiology/neuromuscular fellowship at OSU Medical Center. He is board certified in neurology, clinical neurophysiology, Zachary Simmons, MD electrodiagnostic medicine, and neuromuscular medicine. His Professor of Neurology research interests includes spinal muscular atrophy, muscular Director, Neuromuscular Program dystrophy, myasthenia gravis and other neuromuscular diseases. Pennsylvania State University He is a member of the Muscle Study Group and Project Cure Hershey, Pennsylvania Investigators networks. Dr. Elsheikh received The David Kotlarek Award for Excellence in Compassionate Patient Care and he was Dr. Simmons received his medical degree from the University of awarded by medical students and residents multiple excellence Florida and then trained in neurology at the University of Iowa in teaching awards. He is a member of the American Association and in neuromuscular diseases and electromyography at the of Neuromuscular & Electrodiagnostic Medicine, fellow of the University of Michigan. He now serves as professor of neurology, Royal College of Physicians of Edinburgh (FRCP Edin), and an The Pennsylvania State University, where he is the director of active member of the American Academy of Neurology. the Neuromuscular Program, the Amyotrophic Lateral Sclerosis (ALS) Center, and the Clinical Neurophysiology Laboratory at Pennsylvania State University Hershey Medical Center. David G. Polin, MD Active research programs under his supervision include studies Managing Partner of quality of life in patients with ALS and the development of The EMG Group, Ltd. evidence-based practice protocols for patients with ALS and their Lancaster, Pennsylvania caregivers. Dr. Simmons has served on the American Association of Neuromuscular & Electrodiagnostic Medicine Training Dr. Polin received his undergraduate degree from The Program, Workshop, and Program Committees, has been chair Pennsylvania State University and his medical degree from of the American Board of Electrodiagnostic Medicine (ABEM) Jefferson Medical College in Philadelphia, Pennsylvania. He Maintenance of Certification Committee, and currently is chair of completed residency at New York University in 1987. He is the ABEM Examination Committee.

6 Neuromuscular Complications of Critical Illness Shawn J. Bird, MD Professor of Neurology Director of Neurodiagnostic Laboratories Department of Neurology University of Pennsylvania Philadelphia, Pennsylvania

Bassam A. Bassam, MD Professor of Neurology Director, Neuromuscular Program and EMG Laboratory University of South Alabama Mobile, Alabama

CASE PRESENTATION ONE Examination

History The patient was intubated and on mechanical ventilation. She appeared alert and could follow commands to open and close her A 56-year-old woman with a past medical history of chronic eyes, as well as to look at the examiner. She grimaced appropriately obstructive pulmonary disease was admitted to the intensive care to painful stimuli. Mental status testing was otherwise limited. unit (ICU) with a fever to 36.5°C and bilateral lobar pneumonia. Examination of the cranial nerves was unremarkable. There was Despite intravenous (IV) fluids and broad-spectrum antibiotics, no ptosis and eye movements were intact. There was no facial she developed progressive respiratory failure requiring mechanical weakness. There was moderate neck flexor weakness. There was ventilation. Neurologic examination upon admission was normal. no muscle atrophy or fasciculations in the limbs. Tone was normal. Escherichia coli was cultured from the urine and Enterobacter She was weaker distally (0/5) in the limbs than proximally (2/5 in from the blood. Within the first week she had persistent fever and the legs and 3/5 in the arms). All deep tendon reflexes were absent. tachycardia, requiring vasopressor support, acute She appeared to grimace to painful stimuli over the proximal arm renal insufficiency, leukocytosis (13,500 to 15,200), and hypoxia. and chest, but much less so to painful stimuli applied to the distal She was markedly obtunded with no response to pain and no limbs. voluntary movement. A lumbar puncture and a head computed tomography scan were normal. By hospital day 15, her fever, Initial Differential Diagnosis leukocytosis, and renal insufficiency had resolved. She no longer required vasopressor support and blood and urine cultures were As with most patients who are septic and critically ill, she had a negative. At day 17, she was noted to be more alert and would profound encephalopathy with unresponsiveness. Those patients grimace to painful stimuli. However, she remained profoundly with severe encephalopathy in the setting of sepsis often have weak in her limbs and could not be weaned from mechanical profound limb weakness as well. However, as the critical illness ventilation despite improvement in her respiratory status. improved her encephalopathy largely resolved. Despite that, she remained profoundly weak in her limbs. This profound limb and

7 Neuromuscular Complications of Critical Illness

Table 1. Causes of acute neuromuscular weakness in the critically-ill respiratory muscle weakness was unlikely to be on a central basis patient given her apparent normal mental status, normal brain and spine imaging studies, and the lack of any upper motor neuron signs Spinal cord on her examination. A neuromuscular cause of her weakness was Transverse myelitis suspected. Epidural compression Cord infarction or hemorrhage The list of neuromuscular disorders that may produce weakness Anterior horn cell in the setting of critical illness is extensive (Table 1). However, a Amyotrophic lateral sclerosis and its variants useful way to organize the differential diagnosis of neuromuscular Poliomyelitis or polio-like syndrome weakness in the ICU is to separate pre-existing neuromuscular (e.g., West Nile virus) conditions that produce weakness themselves severe enough to Paralytic rabies necessitate ICU care from those who develop it as a consequence Multiple radiculopathy of critical illness itself. Guillain-Barré syndrome (GBS) and Carcinomatous or lymphomatous meningitis myasthenia gravis (MG) are examples of the former. In this Cytomegalovirus in human immunodeficiency virus particular case, the patient had no neurologic complaints prior to infection hospitalization and her neurologic examination upon admission Peripheral nerve was entirely normal. Therefore, her quadriparesis and respiratory Critical illness * muscle weakness are best explained by a neuromuscular disorder Guillain-Barré syndrome that resulted as a consequence of critical illness. Porphyria Vasculitis The neuromuscular complications of critical illness include Acute massive intoxication (arsenic or thallium) critical illness polyneuropathy (CIP), critical illness myopathy Neuromuscular junction (CIM), and pharmacologically prolonged neuromuscular junction Persistent pharmacologic neuromuscular blockade* blockade (PNJB). She did not receive neuromuscular blocking Myasthenia gravis agents, so that is not a consideration. The distally-predominant Hypermagnesaemia weakness is suggestive of a polyneuropathy, rather than myopathy. Botulism However, this pattern of weakness and reflex loss may be seen in Tick paralysis both disorders and frequently they coexist as discussed below. Organophosphate toxicity Muscle Tests and Procedures Critical illness myopathy* Severe hyopkalemia The serum creatine kinase (CK) was normal at 180 IU/L Periodic paralysis (normal <210). The other blood work was normal and included: Rhabdomyolysis (toxins, infections, or other) electrolytes, calcium, magnesium, glucose, blood urea nitrogen and creatinine, hepatic function studies, complete blood count *Develops as a consequence of, or treatment for, critical (CBC), thyroid stimulating hormone (TSH), sedimentation illness. rate, rheumatoid factor, serum protein electrophoresis, lactate, aldolase, and receptor antibodies. An MRI of the brain and cervical spine was normal. An EEG showed only mild Table 2. Motor nerve conduction studies generalized slowing.

Nerve Amplitude Distal latency Conduction Electrodiagnostic Studies (mV) (ms) velocity (distal/proximal) (m/s) On day 20, an electrodiagnostic (EDX) study was performed. The sensory nerve conduction studies (NCSs) demonstrated absent Median 1.9/1.7 3.8 47 bilateral sural, right median and right ulnar sensory responses. (nl >4.0) (nl <4.4) (nl >50) The radial sensory nerve action potentials (SNAPs) were reduced at 6.2 µV on the right and 4.0 µV on the left (normal >15 µV) Ulnar 1.1/0.9 2.4 50 with a normal velocity of 55 m/s and 57 m/s, respectively (nl >6.0) (nl <3.3) (nl >50) (normal >50 m/s). The motor NCSs were notable for absent or reduced compound motor action potential (CMAP) amplitudes with normal distal latencies and conduction velocities (Table Peroneal NR NR NR 2). This included absent phrenic CMAPs bilaterally. There was Tibial NR NR NR no conduction block or abnormal temporal dispersion. F-wave Phrenic NR NR NR responses were absent from the legs but were normal from the median and ulnar nerves. Repetitive nerve stimulation studies at 2 NR = no response, nl = normal Hz in the right median and spinal accessory nerves were normal. The needle electromyography (EMG) examination was notable

8 Neuromuscular UPDATE II for a sparse amount of abnormal spontaneous activity in the form as many (70%) had electrophysiologic evidence of an axonal of fibrillation potentials. With voluntary effort, the motor unit polyneuropathy. The severity of the neuropathy correlated with potential (MUP) morphology was normal but recruitment was the total time spent in the ICU and with those who survived the markedly reduced, more so distally (Table 3). This EDX study period of critical illness (only half), recovery was as expected was characteristic of an acute, axonal sensory–motor neuropathy. from an acute axonal neuropathy. Those patients who had mild- to-moderate axonal loss recovered fully over months, as a result Discussion of collateral sprouting from remaining motor neurons. Those with severe neuropathy, requiring axonal regeneration for recovery, Critical Illness Polyneuropathy either had no recovery or had a significant persistent deficit.

An axonal sensory–motor polyneuropathy commonly develops Electrophysiologic studies of CIP are those of an axonal in the ICU setting.1,2 This was first described by Bolton and neuropathy.5-10 NCSs are characterized by reduced motor and colleagues3,4 who named it “critical illness polyneuropathy.” sensory response amplitudes. There are no features that suggest During a period of critical illness, characterized by sepsis and demeylination. In general, conduction velocities and distal motor multiorgan failure, their five patients developed a severe sensory– latencies are not significantly affected. Repetitive nerve stimulation motor polyneuropathy. CIP was convincingly shown to be a studies of neuromuscular transmission are unremarkable, unless distal sensory and motor axonal neuropathy, differing from GBS there is persistent pharmacologic neuromuscular blockade. on electrophysiologic and morphologic studies.5 The clinical, Needle EMG examination of a limb muscle often is notable for electrophysiologic, and pathologic features have subsequently spontaneous activity (fibrillation potentials and positive sharp been detailed3,5-7 and, in the setting of critical illness, these waves) with the muscle at rest. With voluntary muscle activation, characteristics define a distinctive form of acute polyneuropathy. there may be an excess of polyphasic MUPs. In significantly weak The clinical features of CIP are similar to other length-dependent muscles, these MUPs are recruited with an increased recruitment neuropathies, with distally-predominant limb weakness and ratio. These features on needle EMG examination are consistent reduced reflexes. Failure to wean from artificial respiration is with acute denervation. Phrenic NCSs often are absent in those common and may be the first recognized manifestation. Muscle with severe neuropathy and needle EMG examination of the atrophy is present, but it is a late finding not seen until the second diaphragm can demonstrate denervation.11 or third month of illness. Sensory loss can be present, but it is usually difficult to demonstrate in patients unable to cooperate Sural nerve biopsy, as well as postmortem autopsy, studies in with the examination due to coexistent encephalopathy, an even patients with CIP show features of an acute, axonal sensory– more common complication of critical illness. If there is reduced motor neuropathy. The pathology is that of axonal degeneration limb movement after painful stimulation of the distal limb and of both sensory and motor fibers without evidence of significant facial grimacing, limb weakness should be suspected. Cranial inflammation or of primary demyelination.3,6,12 DeLetter and nerve involvement is rare and should suggest the possibility of colleagues13 prospectively performed muscle biopsies on 30 another neuromuscular disorder. patients whom they characterized as having CIPNM (critical illness polyneuropathy and myopathy). In these biopsy specimens, One early, illustrative prospective study detailed 43 ICU patients neuropathic changes were seen in 37%, myopathic changes in who had sepsis and multiple organ failure.8 The patients were in 40%, and both in 23%, emphasizing the frequent coexistence of the ICU for a mean of 28 days (range: 5-89) when evaluated and all both CIP and CIM. had evidence of encephalopathy. Thirty-five percent had clinical findings consistent with neuropathy, defined as distal weakness The pathogenesis of CIP is uncertain. As noted above, and hyporeflexia or inability to wean from the respirator. Twice pathological specimens reveal acute primary axonal degeneration

Table 3. Needle electromyography studies

Muscle Fibrillation MUAP MUAP MUAP MUAP potentials duration amplitude polyphasia recruitment

Deltoid 0 NL NL NL ↓ ↓ Biceps 0 NL NL NL ↓ ↓ Triceps 0 NL NL NL ↓ ↓ EDC 0 NL NL NL ↓ ↓ ↓ FDIO 1+ None VM/VL 0 NL NL NL ↓ ↓ TA 0 None PL 1+ None MG 2+ None

EDC = extensor digitorum communis, FDIO = first dorsal interosseous, MG = medial gastrocnemius, MUAP = motor unit action potential, NL = normal, None = no voluntary motor unit potential recruitment, PL = peroneus longus, TA = tibialis anterior, VM/VL = vastus medialis and lateralis, ↓ ↓ = reduced 9 Neuromuscular Complications of Critical Illness of sensory and motor nerve fibers, without inflammation. Stevens and colleagues performed a systematic review of 24 Prospective studies have not supported a causative role of drugs studies (19 prospective) of critically-ill patients who developed (particularly corticosteroids), neuromuscular blocking agents, or CIP and/or CIM.24 Most of these studies avoided the problem of aminoglycoside antibiotics.1,2,6,13-15 No specific toxin, infectious distinguishing between CIP and CIM (see below) by combining agents or nutritional deficiencies have been identified in this them in some fashion as an endpoint. Of the total 1,421 patients disorder. The current view is that cytokines and free radicals in these studies, 655 (46%) developed one or both of these associated with systemic inflammatory response syndrome (SIRS) disorders. adversely affect the microcirculation, producing endoneurial hypoxia and ultimately distal axonal degeneration.16 This view Three large (61-95 patients each) prospective studies have appears to be supported by the finding that critically-ill patients examined risk factors in critically-ill patients for the development with a high Acute Physiology and Chronic Health Evaluation of neuromuscular weakness.20,21,26 All agree that measures of (APACHE) III score and SIRS are most prone to the development illness severity (APACHE III score, presence of SIRS, or organ of CIP.13 failure assessment scores) correlate with the development of CIP/ CIM. The likelihood of developing CIP and/or CIM is strongly Sepsis activates humoral and cellular responses.16 Humoral influenced by the severity of illness. For those with ahigh responses occur locally in tissues as antigen-presenting cells APACHE III score (>85) and the presence of sepsis at the time of produce proinflammatory cytokines such as tumor necrosis factor, study entry (day 4 of mechanical ventilation), the probability of interleukin 1, and free radicals. These humoral factors, together developing CIP/CIM by 30 days was 72%. This compares to only with local cellular responses, interact with adhesion molecules on 8% in patients with low APACHE III scores (<70) and no sepsis.21 platelets and endothelial cells producing platelet-fibrin aggregates This is almost a tenfold higher risk in severely-ill patients. that may reduce capillary flow. Cytokines released in sepsis have histamine-like effects that may increase microvascular The causative association between high-dose corticosteroids, permeability, produce endoneurial edema, and then endoneurial nondepolarizing neuromuscular blocking agents (NMBAs), hypoxia. An increase in local tissue nitric oxide, or endovascular and sedative drugs like propofol with acquired-ICU weakness, relaxing factor, may cause arteriolar dilatation further reducing particularly for CIM, is likely but not established. The first reports capillary flow. The microvascular Bstructures of peripheral nerve of CIM were in patients with status asthmaticus treated with high- lack autoregulation which may make nerve particularly vulnerable dose corticosteroids and NMBAs. Many of the early reports of to these effects.16 critically-ill patients with severe CIM emphasized the prodromal use of corticosteroids and NMBAs.17,18 However, the results Treatment and prognosis. There is no specific pharmacologic from prospective trials have been inconsistent. Of the reports treatment for CIP.17,18 Nevertheless, the first step—recognizing the that detailed this information, there was no significant univariate presence of one of these disorders—often improves management. association with corticosteroids, NMBAs, midazolam, or Prevention of CIP is feasible in part by avoiding risk factors aminoglycosides.24 Multivariable analysis identified a relationship and aggressive medical management of critically-ill patients. between corticosteroids and CIP/CIM in one of the two studies Intensive insulin therapy in ICU patients appears to reduce the that addressed this. In this study, the use of corticosteroids was likelihood of developing CIP and/or CIM. Future treatments of a significant risk factor (odds ratio=14.9).20 Similarly, one of sepsis may further reduce the incidence of these neuromuscular three studies that performed a multivariable analysis showed an consequences of critical illness. association of CIP/CIM with NMBAs (odds ratio=16.32).27 One limitation of the studies that did not show an association was The principles of recovery after CIP are the same as those related the inclusion of a relatively small number of patients who had to axonal loss and recovery from GBS. In muscles where there received substantial doses of corticosteroids and/or NMBAs. is incomplete denervation, recovery of strength occurs with collateral sprouting from the remaining motor axons over a 3 Differentiating Critical Illness Polyneuropathy to 6 month period. More severely affected muscles may require and Critical Illness Myopathy axonal regeneration and reinnervation for recovery. This may take up to 2 years to occur, if ever. As with GBS, the more severe the There is a large cohort of patients in the ICU who have clinical axonal loss the more likely there will be residual disability. The and EDX features common to both CIP and CIM. However, few longterm followup studies on these patients show that many these patients are not easily classified as purely CIP or CIM.17,18 have persistent neurologic deficits.19 The clinical presentation of both disorders is dominated by limb weakness that develops in the ICU, usually accompanied by a Incidence and Risk Factors. In general, the incidence of CIP delay in weaning the patient from mechanical ventilation. and/or CIM appears to be about 50% in those patients who are critically ill in the ICU for more than a week.8,20-24 This has been On electrophysiologic examination, one typically finds features demonstrated in small, but well-designed, single-site prospective common to both disorders. This includes reduced CMAP studies, as well as in larger multicenter ones. Khan and colleagues amplitudes on NCSs and the presence of fibrillation potentials recently conducted a prospective cohort study of patients with on needle EMG examination. Sensory NCSs often are hampered severe sepsis in the ICU.25 Twenty patients survived the analysis by technical factors (limb edema and electrical noise from the period and half (50%) of those developed CIP, CIM, or features ICU equipment), or the sensory responses may be low amplitude of both, most by day 14 of illness. They also found that, of those due to pre-existing neuropathy. Furthermore, the assessment affected, 10% had CIP, 10% had CIM, but 80% had both. of motor unit action potential morphology and recruitment is 10 Neuromuscular UPDATE II often limited by the patient’s encephalopathy or sedation. Direct Table 4. Suggested diagnostic criteria for critical illness polyneuropathy muscle stimulation and measures of the CMAP duration may help identify CIM (see below), but specific diagnostic criteria for 1. The patient is critically ill (sepsis and multiorgan failure, these techniques have not been formally established. Of course, systemic inflammatory response syndrome). establishing the presence of CIM by direct muscles stimulation 2. There is difficulty weaning the patient from mechanical or prolonged CMAP amplitudes does not address the presence ventilator support after nonneuromuscular causes (i.e., or absence of CIP. Despite the limitations in differentiating CIM cardiac and pulmonary disease) have been excluded. from CIP, suggested criteria for each have been proposed (Tables 3. There is possible limb weakness. 4 and 5). 4. There is electrophysiologic evidence of an axonal sensory and motor neuropathy. In addition to the technical considerations that may limit these 5. Under the appropriate clinical circumstances, other causes studies, the risk factors for both disorders overlap (see above) and of acute neuropathy should be excluded (i.e., porphyria, 17.18,25 many patients have a variable combination of both disorders. acute massive intoxications from arsenic or thallium, Nonetheless, is helpful for the ICU staff to recognize that the fulminant vasculitis, etc.). cause of acquired limb weakness and failure to wean in the ICU is due to CIP, CIM, or a combination of both. Adapted from Lacomis.2

CASE PRESENTATION TWO Table 5. Suggested diagnostic criteria for critical illness myopathy* History 1. The patient has had a variable combination of A 46-year-old Caucasian female with history of bronchial asthma nondepolarizing neuromuscular blocking agents, high- and chronic obstructive pulmonary disease was hospitalized dose corticosteroids, and sepsis. for respiratory failure due to status asthmaticus, requiring 2. The patient has the following Clinical features (one or intubations, mechanical ventilation, and transfer to ICU. She was both): paralyzed with vecuronium, and received IV aminophylline and A. Limb weakness. methylprednisolone 500 mg/day. A few days after admission her B. Difficulty weaning from mechanical ventilator blood gases deteriorated, and she was encephalopathic. Chest x-ray support, and cardiac and pulmonary causes have showed left lung pneumonia, and she was treated with antibiotics been excluded. (penicillin and tobramycin). Ten days after admission the patient’s 3. Electrophysiologic studies of the patient show the medical and neurological condition started to improve. Vecuronium following: and methylprednisolone were discontinued, but she continued to A. Reduced motor responses (CMAP amplitudes <80% receive oral prednisone. She was noted to have severe weakness of the lower limit of normal in two or more nerve in all limbs and could not be totally weaned off mechanical without conduction block). ventilator. A neurological consultation was requested. B. Normal repetitive nerve stimulation studies. C. Needle EMG with short-duration, low-amplitude Examination MUPs with early, full, or normal recruitment, with or without fibrillation potentials. The patient was afebrile and her blood pressure was normal. D. Demonstration of muscle inexcitability with direct She was alert and appeared to understand all verbal commands. muscle stimulation techniques. Cranial nerves showed mild facial paresis but were otherwise E. Prolonged CMAP durations (>9.0 ms). normal, including extraocular muscles movements. She was 4. Muscle biopsy demonstrates myopathy with myosin noted to have severe weakness of neck flexors, decreased muscle loss. tone, and severe weakness and mild muscle atrophy of all limbs, more prominent proximally. Tendon reflexes were depressed CMAP = compound muscle action potential, EMG = electromyography, throughout, and she demonstrated normal sensory perception. MUP = motor unit potential *For the clinical diagnosis of critical illness myopathy, patients should have Initial laboratory tests showed elevated serum creatine kinase features 1, 2, 3A, 3B, and one of the following three: 3C, 3D, 3E, or 4. (CK) at 846 IU/L (normal <220). Normal CBC, except for a Adapted from Bird.28 white blood cell count of 9,680 and hematocrit of 33.2%. Serum chemistry profile showed mild hyponatremia and mildly elevated liver , otherwise normal. Her erythrocyte sedimentation rate was 62 mm/h and her TSH was normal.

11 Neuromuscular Complications of Critical Illness Initial Differential Diagnosis phosphatase and periodic acid-Schiff were normal. Electron microscopy showed many fibers with extensive loss of myosin The causes of newly acquired neuromuscular weakness in the thick filaments, with preservation of thin filaments and Z-bands. ICU, noted upon improvement of the underlying critical non- neurological illness, are multiple. Thus the differential diagnosis Discussion is wide including: Critical Illness Myopathy • Patient’s primary disease; • Brain disorders, such as brainstem stroke or CIM is a rapidly evolving myopathy that affects critically ill encephalopathy; patients and may occur independently or in association with CIP. • ICU treatment complications, such as paralyzing agents and Various names to describe this entity have been used, including high-dose corticosteroids; acute quadriplegic myopathy, critical care myopathy, necrotizing • Acute spinal cord or anterior horn cell disorders; myopathy of intensive care, thick filament myopathy, among • Acutely evolving peripheral neuropathy, such as GBS or others. However, the term “critical illness myopathy” is now CIP; widely accepted. • Acute myopathy such as toxic, infectious, rhabdomyolysis, CIM, and a few others; and CIM usually afflicts patients exposed to high-dose IV • Neuromuscular junction disorders, such as exacerbation of corticosteroids, often in combination with nondepolarizing MG or PNJB. neuromuscular junction blocking agents.29 It often occurs in at least one-third of status asthmaticus ICU patients treated with In this case, the patient’s primary critical illness clearly improved high-dose IV corticosteroids and in 7% of organ transplant and no significant metabolic abnormalities were identified. She was patients.30,31 Likewise, CIM also has been associated with not encephalopathic and did not have cranial nerve deficits, long propofol administration, with MG patients who received high- tract signs, or other brainstem findings. Although tendon reflexes dose IV corticosteroids, and, rarely, with patients with sepsis and were depressed, her normal sensory perception did not support the multiorgan failure who were not exposed to corticosteroids.32-34 diagnosis of acutely evolving peripheral neuropathy such as GBS or CIP, and there was no history of exposure to neurotoxic agents. The primary clinical feature is diffuse flaccid muscle weakness that She received vecuronium; however, there were no weaknesses of should have started after the onset of critical illness. The weakness the extraocular muscles, which are key features seen in PNJB. usually is symmetric and proximal more than distal, although in The patient had no prior history of MG, although this may not some patients it is more prominent distally. In most patients all be totally excluded. Given that she had flaccid muscle weakness limb muscles, neck flexors, facial muscles, and the diaphragm (more prominent proximally), muscle atrophy, and a normal are involved. Respiratory failure is common with subsequent sensory perception, acute myopathy was a primary consideration. difficulty to wean from mechanical ventilator. Tendon reflexes are Because she was treated for status asthmaticus with high-dose often depressed, but they can be preserved or increased in some IV methylprednisolone and neuromuscular blocking agents, patients with concurrent encephalopathy. Variable muscle atrophy known risk factors for CIM, this was the most likely diagnosis. is common. Sensory perception usually is normal in cooperative Rhabdomyolysis usually is associated with much higher serum patients. Extraocular muscle weakness rarely is seen in CIM, and CK level than in this case, and often with muscle aches and pain. it usually suggests PNJB.

Electrodiagnostic Evaluation The pathogenesis of CIM is uncertain, although a number of triggering factors are known. Exposure to both corticosteroids and A motor NCS showed normal latencies, conduction velocities neuromuscular-blocking agents suggest a potential pathogenic role, (CVs), F waves, and low-amplitude CMAPs. A sensory NCS however CIM also is reported in patients who were not exposed showed normal latencies, CVs, and SNAPs of the examined to both or either of these agents.34,35 It appears that high-dose nerves. Needle EMG of multiple muscles in various limbs IV corticosteroids in conjunction with neuromuscular blocking showed diffuse fibrillation potentials and positive waves, along agents, critical illness, protracted immobility, high stress catabolic with numerous polyphasic MUPs of low amplitude and short state, and calpain over-expression together trigger apoptosis and a duration. A repetitive motor nerve stimulation test of the median proteolytic mechanism that leads to myofiber atrophy (especially and musculocutaneous nerves showed no clinically-significant type II fibers), intermyofibrillar network disruption, and varying CMAP decrement or increment. degrees of myofiber necrosis and regeneration.29,36-38

Muscle Biopsy Electrophysiologic studies in CIM demonstrate muscle membrane inexcitability to direct electrical stimulus, whereas denervated Muscle biopsy of the left biceps muscle showed myofiber muscle maintains normal excitability. Animal models showed atrophy (especially type II fibers), variation in muscle fibers abnormalities in ion channels and inexcitability of myofibers size, few necrotic and regenerating fibers. Myofibrilar adenosine due to increased channels inactivation at their resting triphosphatase (ATPase) stain showed extensive central pale areas potential.39,40 in most muscle fibers of both histochemical types. Nicotinamide adenine dinucleotide-tetrazolium reductase (NADH-TR) stain Muscle biopsy rarely is needed for diagnosis of CIM, and it shows showed many fibers with myofibrillary network disruption. Acid myofiber atrophy (especially type II fibers), varied degrees of 12 Neuromuscular UPDATE II

A B A B

Figure 1. Muscle biopsy from a patient with critical illness myopathy Figure 2. (A) Electron microscopy of normal muscle biopsy. (B) Electron showing fibers with myosin loss (arrows). (A) ATPase stain pH 9.4. (B) microscopy of muscle biopsy showing loss of thick filaments, with relative ATPase stain pH 4.3. preservation of thin filaments and Z-lines. ATPase = adenosine triphosphatase myofiber necrosis, degeneration, and regeneration. Selective loss Patients with an established or suspected CIM diagnosis should in thick filaments of myosin is a characteristic finding in CIM, be tapered of corticosteroids and paralytic agents. In patients manifesting as a disrupted or patchy loss of myofiber staining with with rhabdomyolysis, IV hydration with alkaline diuresis is ATPase (Fig. 1), and best confirmed by immunohistochemical recommended. stains.37,41 Widespread myofiber necrosis is seen in a more severe entity, known as “acute necrotizing myopathy of intensive In general, most patients with CIM who survive their critical illness care.42 Electron microscopy shows many fibers with myosin recover fully within 2-3 months; however, CIM prolongs the ICU thick filaments loss (Fig. 2). Selective loss of thick filaments in hospitalization and is associated with high medical cost.47 Patients human muscle is not specific to CIM. It has been reported in other with more severe muscle necrosis may have a poor prognosis. disorders, such as dermatomyositis, thrombocytopenic purpura, Mortality secondary to associated critical illness is high, in the cases of congenital myopathy, and myopathy associated with range of 30-50%. human immunodeficiency virus infection.41,43,44 Prolonged Neuromuscular Junction Blockade Laboratory studies in CIM show elevation of serum CK during the first 2 weeks, which can be missed if neuromuscular evaluation is Neuromuscular blocking agents used to facilitate mechanical delayed once medial complications obscure the weakness.45 EDX ventilation may cause a transient prolonged weakness. It usually studies often show reduced CMAP amplitudes, normal NCSs, is seen in patients with renal or liver failure treated with high and the absence of CMAP amplitude changes on repetitive nerve doses of pancuronium or vecuronium.48,49 stimulation. SNAPs usually are normal, but at times they show mild abnormalities suggestive of concurrent neuropathy. Needle Neuromuscular blocking agents are metabolized by the liver EMG examination reveals spontaneous activity, which indicates and cleared by the ; the effect of these agents may last underlying muscle fiber necrosis and membrane irritability. for a number of days, or up to a week or two, after their use Myopathic MUPs are sparse initially but prominent later in the has been discontinued. Concurrent use of corticosteroids and course of the disease.29,46 aminoglycosides may contribute to neuromuscular junction failure. Female gender, acidosis, and hypermagnesemia are risk Treatment and prognosis. There is no specific treatment for CIM factors for PNJB.50,51 Like other ICU neuromuscular weakness available. Treating underlying illness, infections, and associated syndromes, patients with PNJB have generalized flaccid muscle metabolic disorders should be initiated vigorously, along with weakness. However, facial and extraocular muscles weakness adequate nutrition. Muscle biopsy rarely is needed. Prophylaxis and areflexia are characteristic of PNJB. Some patients may have to prevent deep venous thrombosis with subcutaneous heparin or associated CIM, especially those who received concomitant high- enoxaparin (Lovenox®) and pneumatic stockings are warranted. dose corticosteroids.48 Sensory disturbances usually are absent or Physical therapy and nursing care of paralyzed patients are minimal. Electrophysiological studies demonstrate a change in necessary to prevent decubitus, ulcers, and joint contractures the CMAP amplitude and area with repetitive nerve stimulation.52 or superimposed disuse muscle atrophy. Monitoring respiratory Both presynaptic and postsynaptic defects in neuromuscular functions, mechanical ventilation, and early tracheotomy are transmission have been suggested, thus abnormal responses to indicated in patients with respiratory failure. Instituting a low or high stimulation rates have been described. Spontaneous rehabilitation program as patients improve is highly recommended. activity and myopathic MUPs may be seen on needle EMG Avoidance of high-dose IV corticosteroids and cautious use of examination in severe cases.48 Motor and sensory nerve CV and neuromuscular blocking agents (or limiting the duration of their SNAP amplitude are normal. use) reduce the risk of CIM occurrence. Serial CK measurements during high-does IV corticosteroids and paralytic agents in the ICU Prevention of PNJB by minimizing the use (or bolus instead of may provide an early diagnosis of CIM and lower its incidence. continuous administration) of paralytic agents is recommended. 13 Neuromuscular Complications of Critical Illness

Administration of may transiently improve the syndrome: neuromuscular manifestations. Crit Care Med weakness. Hemodialysis is not an effective therapy and only 1996;24:1408-1416. partially reduces the paralytic agents metabolites.53 Hyperglycemia 17. Bird SJ, Rich MM. Critical illness myopathy and polyneuropathy. has a detrimental effect on nerve and muscle function, and Curr Neurol Neurosci Rep 2002;2:527-533. glycemic control is warranted. In general, PNJB is self-limited 18. Bird SJ. Diagnosis and management of critical illness syndrome, usually persisting for days or a week until the paralytic polyneuropathy and critical illness myopathy. Curr Treat Neurol agent metabolites are excreted. PNJB cases with persistent 2007;9:85-92. weakness lasting for more than 2 to 3 weeks are more likely to 19. Guarneri B, Bertolini G, Latronico N. Long-term outcome in have coexisting CIM or CIP.48 patients with critical illness myopathy or neuropathy: the Italian multicentre CRIMYNE study. J Neurol Neurosurg Psychiatry ACKNOWLEDGMENT 2008;79:838-841. 20. De Jonghe B, Sharshar T, Lefaucher JP, et al. Paresis acquired in Discussion of Critical Illness Myopathy of this paper was previous the intensive care unit: a prospective multicenter study. JAMA published this year in: Bassam BA. Neuromuscular disorders acquired 2002;288:2859-2867. in the intensive care unit. In: Bertorini TE, ed. Neuromuscular disorders, 21. De Letter MC, Schmitz PI, Visser LH, et al. Risk factors for the treatment and management. Philadelphia: Elsevier- Saunders; 2011. pp development of polyneuropathy and myopathy in critically ill 162-164. patients. Crit Care Med 2001;29:2281-2286. 22. Lacomis D, Petrella JT, Giuliani MJ. Causes of neuromuscular REFERENCES weakness in the intensive care unit: a study of ninety-two patients. Muscle Nerve 1998;21:610-617. 1. Bolton CF. Neuromuscular manifestations of critical illness. 23. Leijten FSS, Harinck-De Weerd JE, Poortvliet DCJ, De Weerd Muscle Nerve 2005;32:140-163. AW. Critical illness polyneuropathy in multiple organ dysfunction 2. Lacomis D. Neuromuscular disorders in critically-ill patients: syndrome and weaning from the ventilator. Intensive Care Med review and update. J Clin Neuromusc Dis 2011;12:197-218. 1996;22:856-861. 3. Bolton CF, Gilbert JJ, Hahn AF, Sibbald WJ. Polyneuropathy in 24. Stevens RD, Dowdy DW, Michaels RK, et al. Neuromuscular critically ill patients. J Neurol Neurosurg Psychiatry 1984;47:1223- dysfunction acquired in critical illness: a systematic review. 1231. Intensive Care Med 2007;33:1876-1891. 4. Bolton CF. The discovery of critical illness polyneuropathy: a 25. Khan J, Harrison TB, Rich MM, Moss M. Early development of memoir. Can J Neurol Sci 2010;37:431-438. critical illness myopathy and neuropathy in patients with severe 5. Bolton CF, Laverty DA, Brown JD, et al. Critically ill sepsis. Neurology 2006;67:1421-1425. polyneuropathy: electrophysiological studies and differentiation 26. Bednarik J, Vondracek P, Dusek L, et al.Risk factors for critical from Guillain-Barré syndrome. J Neurol Neurosurg Psychiatry illness polyneuromyopathy. J Neurol 2005;252:343-351. 1986, 49:563-573. 27. Garnacho-Montero J, Amaya-Villar R, Garcia-Garmendia JL, et al. 6. Zochodne DW, Bolton CF, Wells GA, et al. Critical illness Effect of critical illness polyneuropathy on the withdrawal from polyneuropathy. Brain 1987;110:819-842. mechanical ventilation and the length of stay in septic patients. Crit 7. Zifko U, Zipko H, Bolton CF. Clinical and electrophysiological Care Med 2005;33:349-354. findings in critical illness polyneuropathy. J Neurol Sci 28. Bird SJ. Critical illness polyneuropathy and critical illness 1998;159:186-198. myopathy. AANEM Critical Care Course. San Diego, California; 8. Witt NJ, Zochodne DW, Bolton CF, et al. Peripheral nerve function 2009. in sepsis and multiple organ failure. Chest 1991;99:176-184. 29. Lacomis D, Giuliani MJ, Van Cott A, Kramer DJ. Acute myopathy 9. Bolton CF Electrophysiologic studies of critically ill patients. of intensive care: clinical, EMG, and pathologic aspects. Ann Muscle Nerve 1987;10:129-135. Neurol 1996;40:645-654. 10. Teener JW, Rich MM, Bird SJ. Other causes of acute weakness in 30. Douglass JA, Tuxen DV, Horne M, Scheinkestel CD, Weinmann the intensive care unit. In: Miller DH, Raps EC, eds. Critical care M, Czarny D, et al. Myopathy in severe asthma. Am Rev Respir neurology. Boston: Butterworth-Heinemann, 1999:69-89. Dis 1992;146:517-519. 11. Bolton CF Clinical neurophysiology of the respiratory system. 31. Perea M, Picon M, Miro O, Orus J, Roig E, Grau JM. Acute Muscle Nerve 1993;16:809-818. quadriplegic myopathy with loss of thick (myosin) filaments 12. Zochodne DW, Bolton CF, Gilbert JJ. Polyneuropathy in critical following heart transplantation. J Heart Lung Transplant illness: pathologic features. Ann Neurol 1985;18:160-166. 2001;20:1136-1141. 13. DeLetter MCJ, Schmitz PIM, Visser LH, et al. Risk factors for 32. Hanson P, Dive A, Brucher J, et al. Acute corticosteroid myopathy the development of polyneuropathy and myopathy in critically ill in intensive care patients. Muscle Nerve 1997;20:1371-1380. patients. Crit Care Med 2001;29:2281-2286. 33. Panegyres PK, Squier M, Mills KR, Newsom-Davis J. Acute 14. Berek K, Margreiter J, Willeit J. in critically ill myopathy associated with large parenteral dose of corticosteroid patients: a prospective evaluation. Intensive Care Med 1993;22:849- in myasthenia gravis. J Neurol Neurosurg Psychiatry 1993;56:702- 855. 704. 15. Druschky A, Herkert M, Radespiel-Troger M, et al. Critical 34. Deconinck N, Van Parijs V, Beckers-Bleukx G, Vanden Bergh illness polyneuropathy: clinical findings and cell culture assay of PV. Critical illness myopathy unrelated to corticosteroids or neurotoxicity assessed by a prospective study. Intensive Care Med neuromuscular blocking agents. Neuromuscul Disord 1998;8:186- 2001;27:686-693. 192. 16. Bolton CF. Sepsis and the systemic inflammatory response 35. Hirano M, Ott BR, Raps EC, et al. Acute quadriplegic myopathy: a 14 Neuromuscular UPDATE II

complication of treatment with steroids, non-depolarizing blocking 45. Hanson P, Dive A, Brucher JM, et al. Acute corticosteroids agents, or both. Neurology 1992;42:2082-2087. myopathy in intensive care patients. Muscle Nerve 1997;20:1371- 36. Di Giovanni SD, Molon A, Broccolini A, et al. Constitutive 1380. activation of MAPK cascade in acute quadriplegic myopathy. Ann 46. Gorson KC, Ropper AH. Generalized paralysis in the intensive care of Neurol 2004;55:195-206. unit: emphasis on the complications of neuromuscular blocking 37. Danon MJ, Carpenter S. Myopathy with thick filament (myosin) agents and corticosteroids. Intensive Care Med 1996;11:219-231. loss following prolonged paralysis with vecuronium during steroid 47. Campellone JV, Lacomis D, Kramer DJ, et al. Acute myopathy treatment. Muscle Nerve 1991;14:1131-1139. after liver transplantation. Neurology 1998;50:46-53. 38. Danon MJ, Kumarasiri M, Etlinger J, et al. Steroid induced 48. Barohn RJ, Jackson CE, Rogers SJ, et al. Prolonged paralysis quadriplegic myopathy with selective thick filament loss: elevated due to nondepolarizing neuromuscular blocking agents and proteasome content suggestive of increased proteolysis of myosin corticosteroids. Muscle Nerve 1994;17:647-654. [abstract]. Neurology 1999;52(suppl 2):A123. 49. Gooch JL. Prolonged paralysis after neuromuscular blockade. 39. Rich MM, Bird SJ, Raps EC, et al. Direct muscle stimulation in Muscle Nerve 1995;18:937-942. acute quadriplegic myopathy. Muscle Nerve 1997;20:665-673. 50. Vanderheyden BA, Reynolds HN, Gerold KB, et al. Prolonged 40. Rich MM, Pinter MJ. Sodium channel inactivation in an animal paralysis after long term vecuronium infusion. Crit Care Med model of acute quadriplegic myopathy. Ann Neurol 2001;50:26- 1992;20:304-307. 33. 51. Segredo V, Caldwell JE, Matthay MA, et al. Persistent paralysis in 41. Showalter CJ, Engel AG. Acute quadriplegic myopathy: analysis critically ill patients after long-term administration of vecuronium. of myosin isoforms and evidence for calpain-mediated proteolysis. N Engl J Med 1992;327:524-528. Muscle Nerve 1997;20:316-322. 52. Gooch JL, Moore MH, Ryser DK. Prolonged paralysis after 42. Zochodne DW, Ramsay DA, Saly V, Shelley S, Moffatt S. Acute neuromuscular junction blockade: case reports and electrodiagnostic necrotizing myopathy of intensive care: electrophysiological findings. Arch Phys Med Rehabil 1993;74:1007-1011. studies. Muscle Nerve 1994;17:285-292. 53. Segredo V, Matthay MA, Sharma ML, et al. Prolonged neuromuscular 43. Yarom R, Sphira Y. Myosin degeneration in a congenital myopathy. blockade after long-term administration of vecuronium in two Arch Neurol 1977;34:114-115. critically-ill patients. Anesthesiology 1990;72:566-570. 44. Simpson DM, Bender AN. Human immunodeficiency virus- associated myopathy: Analysis of 11 patients. Ann Neurol 1988;24:79-84.

15 16 Alternative and Rehabilitative Approaches to Managing Neuromuscular Disease

Sunil K. Aggarwal, MD, PhD Resident Physical Medicine and Rehabilitation Rusk Institute of Rehabilitation Medicine New York University New York, New York

Gregory T. Carter, MD, MS Clinical Professor of Rehabilitation Medicine Department of Clinical Neurosciences Providence Medical Group Olympia, Washington

INTRODUCTION The Western practitioner may need to change his or her expected patient outcomes from what is commonly anticipated in Western Alternative health therapies may include herbal and folk medicines, medicine. Alternative therapies may help improve the quality homeopathy, dietary manipulation, naturopathy, chiropractic, of life (QOL) in NMD, helping the patient feel better, enjoy an and many forms of mind body medicine, including meditation, improved overall QOL, and provide the patient with a sense of hypnotherapy, music therapy, and many other approaches (as this control. Alternative therapies may help via reducing anxiety, is not meant to be an exhaustive list). Alternative therapies are stress, depression, sleeplessness, and tension. This may help to employed as an alternative to conventional medicine in the hope reduce feelings of pain, breathlessness, constipation, diarrhea, that they exert some healing process on the underlying disease. tiredness, and lack of appetite as well as help to moderate some of Moreover, what was considered alternative care 20 years ago may the effects of NMD, including muscle cramping or spasms. now be considered mainstream medicine. Many alternative health therapies and nonpharmacological modalities now are being used It is important that the patient is aware that these treatments are routinely in western medicine. Mainstream medical treatment not an alternative to traditional, orthodox medical approaches. and alternative or complementary therapy today are considered Rather, they should be viewed as complementary therapies part of the same spectrum of integrated medicine. There are now that provide emotional, spiritual, and physical well being, in some very strong scientifically valid studies verifying the safety conjunction with what may be otherwise used by the clinician. and efficacy of alternative therapies. Indeed, nontraditional and For patients with NMDs, therapies that have been tried include nonpharmaceutical based approaches may have some distinct aromatherapy, acupuncture, homeopathy, massage, hypnotherapy, advantages over conventional medicine. Recent studies showed herbal therapy, reiki, meditation, and reflexology. It is critical that that approximately 72 million adults in the United States used physicians consider that their NMD patients may be availing complementary and alternative therapies, spending an out-of- themselves of alternative therapies and it is critical to double pocket amount of over a billion dollars annually.1-3 In most check that any additional alternative treatment will not adversely neuromuscular diseases (NMDs), where there is a lack of affect any current treatment program. effective treatment, it is necessary to explore and evaluate such therapies.4,5 This two-part discussion will focus on the emerging role of herbal cannabis therapeutics in the management of NMD as well as review the role of exercise in helping maintain and improve QOL for this patient population. 17 Alternative and Rehabilitative Approaches to Managing Neuromuscular Disease MEDICAL MARIJUANA (CANNABIS) AND The fact that both CB1 and CB2 receptors have been found on NEUROMUSCULAR DISEASE immune cells suggests that cannabinoids play an important role in the regulation of the immune system. Recent studies show that A Brief Overview of Cannabinoid Pharmacology cannabinoids downregulate cytokine and chemokine production, both mechanisms that suppress inflammatory responses.32-35 The use of marijuana (heretofore referred to as cannabis) Manipulation of endocannabinoids (i.e., via use of exogenous as medicine continues to grow in the United States as more cannabis) has great potential treatment viability against healthcare providers become educated about the physiologic inflammatory disorders, including the inflammation seen inthe importance of the endogenous cannabinoid system and about the central nervous system (CNS) of patients with amyotrophic lateral wide safety margins and broad clinical efficacies of cannabinoids, sclerosis (ALS).The potential use of cannabinoids as a novel class the active ingredients in cannabis. Cannabinoid medicines are of anti-inflammatory agents may become one the predominant available in both purely botanical (natural) and purely chemical indications, as it addresses not only neuromodulation but pain as varieties. Prior to the last decade there was little known about well.36,37 Indeed, any number of inflammatory processes that are the specific pharmacological and molecular effects of cannabis. at least partially triggered by activated T cells or other cellular However, important advances have taken place recently that have immune components could be treated with cannabis and other greatly increased the understanding of the receptors and ligands cannabinoid-based medicines. composing the endogenous cannabinoid system.6-18 Research has shown that two major cannabinoid receptor subtypes exist, Tamoxifen, a Food and Drug Administration-approved drug used including the cannabinoid receptor type 1 (CB1) subtype, which to treat breast cancer, is a terpene, the same chemical classification is predominantly expressed in the brain and nervous system, and as the cannabinoids.38-40 Terpenes are organic, lipid soluble the cannabinoid receptor type 2 (CB2) subtype, which is primarily compounds which, like petrochemicals, readily penetrate the found on the cells of the immune system.19-21 A variety of ligands highly lipophilic CNS.38 This affinity for neural tissue may explain for these receptors based on the cannabinoid structure have been the potential implication of prior petrochemical exposure as a risk synthesized and studied. Experiments performed with several factor for ALS. The chemical similarity between cannabinoids types of neural cells that endogenously express the CB1 receptor and tamoxifen points to a possible shared mechanism of action suggest that the activation of protein kinases may be responsible for neural protection.40 Phase II clinical trials of tamoxifen in for some of the cellular responses elicited by the CB1 cannabinoid ALS demonstrated preliminary efficacy and safety.39 Results from receptor.22 a Phase IIB study demonstrated an increase in survival by the end of 2 years in the groups taking the higher doses of tamoxifen, The discovery of the endocannabinoids (i.e., endogenous although two lower dose groups had no increase in survival.39 The metabolites capable of activating the cannabinoid receptors) and three higher dose groups experienced a 4 to 6 month prolongation the understanding of the molecular mechanisms leading to their of survival over a 24-month trial, with no significant side effects biosynthesis, release, and inactivation have created a new area in observed.39 Interestingly, glutamate uptake in cultured retinal cells research on the pharmaceutical applications of cannabinoid-based is inhibited by tamoxifen, thus this mechanism may be part of a medicines.23 The characterization of endocannabinoids such as possible beneficial effect in ALS.38 anandamide and the detection of widespread cannabinoid receptors in the brain and peripheral tissues suggest that the cannabinoid The Cannabis Plant system represents a previously unrecognized ubiquitous network in the nervous system. Cannabis is a remarkably complex plant. There are several existing phenotypes, with each containing over 400 distinct chemical Cannabinoid receptors are G protein-coupled, seven-segment moieties. Approximately 85 are chemically unique and classified transmembrane proteins similar to the receptors of other neu- as plant cannabinoids or phytocannabinoids.41-44 The cannabinoids rotransmitters such as dopamine, serotonin, and norepinephrine.22,23 are 21 carbonmonoterpenoids, biosynthesized predominantly via Dense receptor concentrations are found in the cerebellum, basal a recently discovered deoxyxylulose phosphate pathway.41 The ganglia, and hippocampus, likely accounting for the effect of ex- cannabinoids are lipophilic. Delta-9 (THC) ogenously administered cannabinoids on motor tone and coordi- and delta-8 THC appear to produce the majority of the psychoactive nation as well as mood state.24,25 Low concentrations are found effects of cannabis.45,46 Delta-9 THC, the active ingredient in in the brainstem, accounting for the low potential for lethal over- dronabinol (Marinol), is the most abundant cannabinoid in the dose with cannabinoid-based medicines.26-30 A growing number plant and this has led researchers to hypothesize that it is the main of strategies are emerging for separating sought after therapeu- source of the drug’s impact. tic effects of cannabinoid receptor agonists from the unwanted consequences of CB1 receptor activation. Recently, ligands have However, other major plant cannabinoids, including cannabidiol been developed that are potent and selective agonists for CB1 and (CBD) and cannabinol (CBN), may modify the pharmacology of CB2 receptors, as well as potent CB1 selective antagonists and THC and have distinct effects of their own. CBD is the second inhibitors of endocannabinoid uptake or .31 In addi- most prevalent of cannabis’s active ingredients and may produce tion, varieties of cannabis are known to contain a mix of partial most of its effects at moderate, midrange doses. Up to 40% of cannabinoid agonists and antagonists, which can be rationally the cannabis resin in some strains is CBD.43 The amount varies utilized. This knowledge may lead to the design of synthetic can- according to plant. Some varieties of Cannabis sativa have been nabinoid agonists and antagonists as well as cannabis strains with found to have no CBD.43 CBD appears to modulate and reduce high therapeutic potential. any untoward effects of THC.45 It has significant anticonvulsant, 18 sedative, and other pharmacological activities likely to interact Neuromuscular UPDATE II with the effects of THC.46-49 CBD may induce sleep and may Bilsland and colleagues showed that treatment of postsymptomatic, provide some protection against seizures for epileptics.49 Both 90-day-old G93A-SOD1 mice with a synthetic cannabinoid THC and CBD are derived from a common precursor known as (WIN55,212-2) significantly delayed disease progression.57 cannabigerol. THC over time breaks down into CBN, which has Furthermore, genetic ablation of the fatty acid amide hydrolase fewer therapeutic properties. (FAAH) , which results in raised levels of the endocannabinoid anandamide by preventing its breakdown, How Might Cannabis Help in Neuromuscular prevented the appearance of disease signs in 90-day-old G93A- Disease: A Look at Amyotrophic Lateral Sclerosis SOD1 mice. Surprisingly, elevation of cannabinoid levels with either WIN55, 212-2 or FAAH ablation had no effect on life span. It is now known that during active neurodegeneration from Ablation of the CB1 receptor, in contrast, had no effect on disease disease or trauma in the CNS, the concentration of tumor necrosis onset in G93A-SOD1 mice but significantly extended life span. factor alpha (TNF-α) rises well above normal levels during the Together these results indicate that cannabinoids have significant inflammatory response. Addition of exogenous TNF-α, both neuroprotective and disease-modifying effects in this model of in vitro and in vivo, to neurons has been shown to significantly ALS and suggest that these beneficial effects may be mediated by potentiate glutamatergic excitotoxicity. Thus the discovery of non-CB1 receptor-based mechanisms. drug targets reducing excess TNF-α expression may help protect neurons after injury. Zhao and colleagues investigated the The Role of the Endocannabinoid System neuroprotective role of the CB1 receptor after TNF-α exposure in the presence or absence of CB1 agonists.50 They demonstrated The endocannabinoid anandamide demonstrates dopamine- that CB1 activation blocks the TNF-α–induced increase in blocking and anti-inflammatory effects and is also tonically inflammation, thus protecting the neurons from damage. Thus, active in the periaqueductal gray matter.55 Endocannabinoids also neuroprotective strategies which increase CB1 activity may help modulate glutamatergic neurotransmission indirectly via NMDA to reduce damage to motor neurons in ALS that are mediated by (N-methyl D-aspartate) receptors, and these pathways can be CNS inflammation. modulated to produce a clinical effect, such as reduction in motor tone, increase in seizure threshold, protection from neuronal injury, Additionally, CB2 receptors are dramatically upregulated in decrease in perception of pain, and elevation in mood state.56-66 inflamed neural tissues associated with CNS disorders, including These clinical, biochemical, and pathophysiological patterns ALS.51 In mutant mice expressing the glycine to alanine could reflect an underlying abnormality in the endocannabinoid substitution of cytosolic Cu, Zn-superoxide dismutase (G93A- system in ALS that potentially could be treated with exogenous SOD1), endogenous cannabinoids are elevated in the spinal cords cannabinoids (i.e., via clinical use of cannabis or some derivative of symptomatic mice.51 Furthermore, treatment with nonselective thereof). cannabinoid partial agonists prior to, or upon, symptom appearance minimally delays disease onset and prolongs survival through Using Cannabis to Manage Clinical Symptoms of undefined mechanisms. Shoemaker and colleagues demonstrated Neuromuscular Disease that messenger RNA levels, receptor binding, and function of CB2, but not CB1, receptors are dramatically and selectively In addition to the neuroprotective effect, patients report that upregulated in spinal cords of G93A-SOD1 mice in a temporal cannabis helps in treating symptoms of the disease, including pattern paralleling disease progression.52 Daily injections of the alleviating pain and muscle spasms, improving appetite, selective CB2 agonist AM-1241, initiated at symptom onset, diminishing depression, and helping to manage sialorrhea increased the survival interval after disease onset by 56%.52 (excessive drooling) by drying up saliva in the mouth.67-69 Indeed, in a large survey it was noted that ALS patients who were able to The primary murine model for human ALS is the G93A-SOD1 obtain cannabis found it preferable to prescription medication in mutant mouse, which is genetically engineered to replicate managing their symptoms. However, this study also noted that the familial ALS.53 There is strong evidence in this model that the biggest reason ALS patients were not using cannabis was their endocannabinoid system is involved, both directly and indirectly, inability to obtain it, either due to legal or financial reasons or lack in the pathophysiology of the disease. Several recent studies have of safe access.67 highlighted this. Rossi and colleagues investigated both excitatory and inhibitory synaptic transmission in the striatum of symptomatic There are many other clinical problems faced by NMD patients G93A-SOD1 ALS mice, along with the sensitivity of these synapses that could be helped by cannabis. The majority of NMD patients to CB1 receptor stimulation.54 They reported a reduced frequency experience significant pain.70-94 The pain largely is due to of glutamate-mediated spontaneous excitatory postsynaptic immobility, which can cause adhesive capsulitis, mechanical back currents and increased frequency of gamma-amino-butyric acid pain, pressure areas on the skin, and, more rarely, neuropathic (GABA)-mediated spontaneous inhibitory postsynaptic currents pain.78,80,84 Pain in NMD is a frequent symptom especially in the in recordings from striatal neurons in ALS mice. This likely is due later stages of disease and can have a pronounced influence on to some presynaptic defects in transmitter release. The sensitivity QOL and suffering.82,83,94 Treatment of pain therefore should be of CB1 receptors in controlling both glutamate and GABA recognized as an important aspect of palliative care in NMD. transmission was potentiated in ALS mice. This provides good Despite the major pain problems encountered by patients with evidence that adaptations of the endocannabinoid system might be NMD, there are no clear guidelines and few randomized clinical involved in the pathophysiology of ALS. This is consistent with trials that address how to manage pain in this population. However, current theories on pathophysiological mechanisms of ALS.55,56 as noted previously, the cannabinoids have been shown to produce 19 Alternative and Rehabilitative Approaches to Managing Neuromuscular Disease an anti-inflammatory effect by inhibiting the production and to desired effect” and an individual, patient-controlled, dosing action of TNF-α and other acute phase cytokines.6 Additionally, model may be used. Dosing paradigms for clinical effects in terms cannabis may reduce pain sensation, likely through a brainstem of pain have been previously described in the literature.111,112 A circuit that also contributes to the pain suppressing effects of patient-determined, self-titrated dosing model is acceptable given morphine.95-97 Cannabinoids produce analgesia by modulating the low toxicity of cannabis and the multiple variables involved rostral ventromedial medulla neuronal activity in a manner similar here. However, based on the available studies, a typical pulmonary to, but pharmacologically distinct from, that of morphine.95 This administration dosing range would likely be 1-2 g/day of cannabis analgesic effect is also exerted by some endogenous cannabinoids with an average THC content of 20% by weight.111,112 (anandamide) and synthetic cannabinoids (methanandamide) and may be prevented by the use of selective antagonists.98-100 Thus NONPHARMACOLOGICAL/REHABILITATION cannabinoids are centrally and peripherally acting analgesics with STRATEGIES TO IMPROVE HEALTH-RELATED a different mechanism of action than opioids, although the analgesia QUALITY OF LIFE IN PATIENTS WITH produced by cannabinoids and opioids may involve similar and NEUROMUSCULAR DISEASE synergistic pathways at the brainstem level. A recent systematic review and meta-analysis of double-blind randomized controlled There is incredible diversity in the spectrum of disease burden trials that compared any cannabis preparation to placebo among incurred by individuals with NMD. However, subjects with chronic pain showed a total of 18 completed trials. weakness is the ultimate cause of the majority of clinical problems The studies indicate that that cannabis is moderately efficacious in NMDs. There have been a number of well-controlled studies for treatment of chronic pain.101 In the setting of advanced documenting the effect of exercise as a means to gain strength NMD, the medications should be titrated to the point of comfort. in NMDs, although much remains to be learned.113-124 In slowly Concomitant use of narcotics may be needed if pain is severe. progressive NMDs, a 12-week moderate resistance (30% of However, cannabis may lower the amount of opiates needed, maximum isometric force) exercise program resulted in strength and because cannabinoids do not cause respiratory depression gains ranging from 4-20% without any notable deleterious or constipation, this may be very advantageous. The anti-emetic effects.122 However, in the same population, a 12-week high- effect of cannabis may help with the nausea sometimes associated resistance (training at the maximum weight a subject could lift with narcotics. 12 times) exercise program showed no further added beneficial effect compared with the moderate resistance program, and there In addition to pain, spasticity is also a major problem for some was evidence of overwork weakness in some of the subjects, NMD patients, particularly those with ALS. Spasticity is induced particularly following eccentric contractions.125,126 both at the motor cortex and at the spinal cord level through the loss of motor neuron inhibition.102-106 Cannabis has an inhibitory In a study comparing patients with Charcot-Marie-Tooth (CMT) effect via augmentation of GABA pathways in the CNS.105 This disease and to patients with myotonic muscular dystrophy (MMD), produces motor neuron inhibition at spinal levels in mice. Several only the CMT patients appeared to benefit significantly from a past studies have suggested that cannabinoid therapy provide at strengthening program.127 This clearly points out that the most least a subjective reduction of spasticity, although virtually all effective exercise regimens for neuropathies and myopathies most of the studies have been done in patients with multiple sclerosis likely are going to be different, although further investigation is (MS).106,107 One survey study has shown that ALS patients do needed. In rapidly progressive disorders like Duchenne muscular subjectively report that cannabis helps alleviate symptoms of dystrophy (DMD) and ALS, there is active ongoing muscle spasticity.67 As mentioned previously, cannabis is also a potent degeneration and the risk for overwork weakness and exercise- antisalivatory compound and ALS patients have reported benefit induced muscle injury is much greater. In this population exercise in controlling saliva when using it.67 Although this remains to should be prescribed with caution and a common sense approach. be studied, other potential uses of cannabis in managing NMD It is advisable that all NMD patients be advised not to exercise to symptoms include, improving appetite, mood state, and sleep exhaustion, due to the risk of exercised-induced muscle damage. patterns. NMD patients in an exercise program should be monitored for signs of overwork weakness. This includes excessive delayed How Would an Neuromuscular Disease Patient onset muscle soreness. This usually occurs 24-48 hours following Safely Use Cannabis as Medicine? exercise. Other warning signs include severe muscle cramping, heaviness in the extremities, and prolonged dyspnea. Cannabinoids are volatile and will vaporize at temperatures in the range of 200°F, much lower than actual combustion.108-110 Heated Submaximal, low-impact aerobic exercise (walking, swimming, air can be drawn through cannabis and the active compounds will stationary bicycling) will improve symptoms of fatigue via vaporize, which can then be inhaled. This delivers the cannabinoids enhancement of cardiovascular performance and increase muscle in a rapid manner that can be easily titrated to desired effect.108-110 oxygen and substrate utilization.127-129 This is important because Additionally, cannabis can be ingested orally or through a feeding fatigue is a significant limiting factor in physical performance tube using extracts prepared in lipophilic or alcohol-based media, in patients with NMDs.130,131 Fatigue in this setting likely is although absorption is much slower, making dose titration more multifactorial, due to decondtioning and impaired muscular difficult. activation.132 Improving cardiopulmonary performance through aerobic exercise will improve not only physical functioning For patients with severe dysphagia, inhalation offers obvious but also improve mood state and help fight depression and advantages. Dosing for symptom management would be “titrate osteoporosis, which in turn reduces fracture risk.132,133 Patients with 20 Neuromuscular UPDATE II NMD have been noted to have higher scores on the subprofiles joint instability. The decision to brace should include the risk of indicating depression on the Minnesota Multiphasic Personality added weight of the brace and the willingness of the patient to Inventory.134 Aerobic exercise also will help achieve and maintain use the brace. NMD patients should be referred for a course of ideal body weight, and improve pain tolerance. Nonballistic, physical therapy after being fitted with braces to help them learn sustained muscle stretching also is helpful and should be done to use the devices effectively. routinely after exercise. Equipment Orthotics Proper equipment can significantly improve quality of life for Joint contractures and scoliosis are a major clinical problem in an NMD patient. Common examples include hospital beds, NMDs, particularly DMD and spinal muscular atrophy type II commode chairs, wheelchairs and wheelchair ramps, hand-held patients.135 Routine examination of the spine and major joints in showers, bathtub benches, grab bars, and raised toilet seats. An NMD patients should be performed at each clinic visit. Contractures occupational therapist is best qualified to determine if any of these appear to be related to prolonged static limb positioning and devices would be useful for the NMD patient. frequently develop shortly after the patient becomes wheelchair dependent.23 In ambulatory patients, upper extremity contractures Wheelchairs are a critical component of mobility in those with may occur and be complicated by joint subluxation, particularly severe NMD. Wheelchairs need to be fitted appropriately with in the shoulder girdle. Slings may provide support but will not the right frame size, type of seat, lumbar support and cushioning prevent contracture formation. Again, stretching and positional to avoid pressure ulcers. Other mechanical devices, such as splinting may slow the progression of contractures, although the the Tilt-N-Space (Postural Seating Materials, Inc., Lawrence, actual efficacy of this has not been well studied or documented Kansas) allow the patient to independently tilt the wheelchair in the literature. Surgical release of contractures in the lower seat, providing improved comfort and better pressure relief for the extremities may allow a patient to be functionally braced. This skin. These devices often can be retrofitted on to existing chairs. may prolong ambulation although a number of studies have The patient should be evaluated by a physical or occupational shown that weakness, not contractures, contribute most to the loss therapist to ensure proper wheelchair prescription. Simply giving of functional ambulation.135-137 the patient a prescription for a wheelchair frequently results in a chair that does not fit properly or has improper components.138-139 For the extremities, bracing should be used with the goal of Power wheelchairs are indicated in most NMD patients who can no improving function and joint stability. Long-leg bracing to longer ambulate and do not have enough upper extremity strength prolong ambulation time in DMD has been one of the best-studied to independently propel a manual chair. Although expensive, uses in NMDs. A number of studies have shown that ambulatory power wheelchairs can be justified to third-party payers on the ability may be prolonged up to 2 years with long-leg braces and basis that they help prolong independent mobility, thus decreasing appropriate contracture release. However, it is not clear if this medical and psychological comorbidity. represents a subset of patients with a slower disease progression and relatively less weakness.136 Further, there does not appear to For patients who can still ambulate, walkers or quad (four-point) be any clear association between prolonging ambulation with canes help reduce fall risk. Pressure-relieving mattresses, with long-leg bracing and delaying or decreasing scoliosis in DMD. foam wedges for proper positioning, help prevent pressure skin If bracing is used, a “long-leg brace” or knee-ankle-foot orthosis ulcers. In some NMD patients, particularly those with ALS, severe (KAFO) generally is needed due to the amount of weakness weakness in neck musculature may produce neck pain and muscle in hip and knee extension as well as ankle plantar flexion and spasms. A cervical collar, particularly the Freeman or Headmaster dorsiflexion.126 type (a wire-frame collar with padding over the pressure points) may be very helpful. In patients with dysarthria, typically ALS Most CMT patients require “short-leg braces” or ankle-foot patients, augmentative communicative aids, including an alphabet orthoses (AFOs). It is best if these are custom-made with a board, word board, or computer-based speech synthesizer, lightweight polymer (polypropylene or carbon fiber). They should can maintain functional communication. A speech language fit intimately to avoid skin problems and provide good stability. If pathologist is best qualified to determine which, if any, of these a pressure sores occur, the patient should be taken out of the brace devices would work best. until they heal. Double metal upright AFOs may be built into the shoe but are usually too heavy and may limit ambulation for those DEDICATION patients with proximal muscle weakness. If there is significant ankle instability noted, the braces should be high profile (come Dedicated with loving respect to the memory of Drs. David D. Kilmer and around in front of the malleoli). Pes cavus and hammertoe Lisa S. Krivickas. Their pioneering work in this area is cited frequently deformities can be accommodated with built-up arches and herein. metatarsal bars. CMT and other sensory neuropathy patients are at very high risk for skin ulcers and neuropathic arthritis (Charcot REFERENCES joint). Thus, skin integrity and joint stability should be checked at every clinic visit. 1. Avorn J. Post-modern drug evaluation. The deconstruction of evidence-based regulation. Pharmacoeconomics 2000;18 Suppl Patients with NMD weakness may benefit from bracing, 1:15-20. depending on the distribution of weakness, gait problems, and 21 Alternative and Rehabilitative Approaches to Managing Neuromuscular Disease

2. Tindle HA, Wolsko P, Davis RB, Eisenberg DM, Phillips RS, 19. Maenpaa H, Mannerstrom M, Toimela T, Salminen L, Saransaari P, McCarthy EP. Factors associated with the use of mind body Tahti H. Glutamate uptake is inhibited by tamoxifen and toremifene therapies among United States adults with musculoskeletal pain. in cultured retinal pigment epithelial cells. Pharmacol Toxicol Complement Ther Med 2005 Sep;13(3):155-164. 2002;91(3):116-122. 3. Tindle HA, Davis RB, Phillips RS, Eisenberg DM. Trends in use of 20. Klein TW, Lane B, Newton CA, Friedman H. The cannabinoid complementary and alternative medicine by US adults: 1997-2002. system and cytokine network. Proc Soc Exp Biol Med Altern Ther Health Med 2005;11(1):42-49. 2000;225(1):1-8. 4. Hainline B. Neuropathic pain: mind-body considerations. Neurol 21. Pertwee RG. Cannabinoid receptor ligands: clinical and Clin 2011;29(1):19-33. neuropharmacological considerations, relevant to future drug 5. Dubowitz V.Current concerns in the therapy of neuromuscular discovery and development. Expert Opin Investig Drugs disorders. Neuromuscul Disord 2011;21(1):3. 2000;9(7):1553-1571. 6. Abood ME. Molecular biology of cannabinoid receptors. Handb 22. Akinshola BE, Chakrabarti A, Onaivi ES. In vitro and in vivo Exp Pharmacol 2005;168:81-115. action of cannabinoids. Neurochem Res 1999;24(10):1233-1240. 7. McAllister SD, Hurst DP, Barnett-Norris J, Lynch D, Reggio 23. Di Marzo, Bisogno T, De Petrocellis L. Endocannabinoids: new PH, Abood ME, Structural mimicry in class A G protein-coupled targets for drug development. Curr Pharm Des 2000;6(13):1361- receptor rotamer toggle switches: the importance of the F3.36(201)/ 1380. W6.48(357) interaction in cannabinoid CB1 receptor activation. J 24. Hanuš LO, Mechoulam R. Novel natural and synthetic ligands of Biol Chem 2004;279(46):48024-48037. the endocannabinoid system. Curr Med Chem 2010; Feb 18. [Epub 8. McAllister SD, Chan C, Taft RJ, Luu T, Abood ME, Moore DH, ahead of print] Aldape K, Yount G, Cannabinoids selectively inhibit proliferation 25. De Laurentiis A, Fernández Solari J, Mohn C, Zorrilla Zubilete M, and induce death of cultured human glioblastoma multiforme cells. Rettori V. Endocannabinoid system participates in neuroendocrine J Neurooncol 2005;74(1):31-40. control of homeostasis. Neuroimmunomodulation 2010;17(3):153- 9. Kapur A, Samaniego P, Thakur GA, Makriyannis A, Abood ME. 156. Mapping the structural requirements in the CB1 cannabinoid 26. Nagarkatti P, Pandey R, Rieder SA, Hegde VL, Nagarkatti M. receptor transmembrane helix II for signal transduction. J Cannabinoids as novel anti-inflammatory drugs. Future Med Chem Pharmacol Exp Ther 2008;325(1):341-348. 2009;1(7):1333-1349. 10. Gehani NC, Nalwalk JW, Razdan RK, Martin BR, Sun X, Wentland 27. Mouslech Z, Valla V. Endocannabinoid system: An overview of M, Abood ME, Hough LB. Significance of cannabinoid CB1 its potential in current medical practice. Neuro Endocrinol Lett receptors in improgan antinociception. J Pain 2007;8(11):850-860. 2009;30(2):153-179. 11. Anavi-Goffer S, Fleischer D, Hurst DP, Lynch DL, Barnett-Norris 28. Wegener N, Koch M. Neurobiology and systems physiology of J, Shi S, Lewis DL, Mukhopadhyay S, Howlett AC, Reggio the endocannabinoid system. Pharmacopsychiatry 2009;42(Suppl PH, Abood ME, Helix 8 Leu in the CB1 cannabinoid receptor 1):S79-S86. contributes to selective signal transduction mechanisms. J Biol 29. De Petrocellis L, Di Marzo V. An introduction to the endocannabinoid Chem 2007;282(34):25100-25113. system: from the early to the latest concepts. Best Pract Res Clin 12. McAllister SD, Rizvi G, Anavi-Goffer S, Hurst DP, Barnett-Norris Endocrinol Metab 2009;23(1):1-15. J, Lynch DL, Reggio PH, Abood ME, An aromatic microdomain 30. Goodfellow CE, Glass M. Anandamide receptor signal transduction. at the cannabinoid CB(1) receptor constitutes an agonist/inverse Vitam Horm 2009;81:79-110. agonist binding region. J Med Chem 2003;46:24:5139-5152. 31. Pertwee RG. Ligands that target cannabinoid receptors in the brain: 13. McAllister SD, Tao Q, Barnett-Norris J, Buehner K, Hurst DP, from THC to anandamide and beyond. Addict Biol 2008;13(2):147- Guarnieri F, Reggio PH, Nowell Harmon KW, Cabral GA, Abood 159. ME, A critical role for a tyrosine residue in the cannabinoid receptors 32. Cencioni MT, Chiurchiù V, Catanzaro G, Borsellino G, Bernardi G, for ligand recognition. Biochem Pharmacol 2002;63(12):2121- Battistini L, Maccarrone M. Anandamide suppresses proliferation 2136. and cytokine release from primary human T-lymphocytes mainly 14. Abood ME, Rizvi G, Sallapudi N, McAllister SD, Activation of the via CB2 receptors. PLoS One 2010;5(1):e8688. CB1 cannabinoid receptor protects cultured mouse spinal neurons 33. Pasquariello N, Catanzaro G, Marzano V, Amadio D, Barcaroli against excitotoxicity. Neurosci Lett 2001;309(3):197-201. D, Oddi S, Federici G, Urbani A, Finazzi Agrò A, Maccarrone M. 15. Panikashvili D, Simeonidou C, Ben Shabat S, Hanus L, Breuer A, Characterization of the endocannabinoid system in human neuronal Mechoulam R, Shohami E. An endogenous cannabinoid (2 AG) cells and proteomic analysis of anandamide-induced apoptosis. J is neuroprotective after brain injury. Nature 2001;413(6855):527- Biol Chem 2009;284(43):29413-29426. 531. 34. Pandey R, Mousawy K, Nagarkatti M, Nagarkatti P. 16. Chen Y, Buck J. Cannabinoids protect cells from oxidative cell Endocannabinoids and immune regulation. Pharmacol Res death: a receptor independent mechanism. J Pharmacol Exp Ther 2009;60(2):85-92. 2000;293(3):807-812. 35. Parolaro D, Massi P, Rubino T, Monti E. Endocannabinoids in the 17. Eshhar N; Striem S; Biegon A. HU 211, a non psychotropic immune system and cancer. Prostaglandins Leukot Essent Fatty cannabinoid, rescues cortical neurones from excitatory amino acid Acids 2002;66(2-3):319-332. toxicity in culture. Neuroreport 1993;5(3):237-240. 36. Mato S, Alberdi E, Ledent C, Watanabe M, Matute C. CB1 18. Guzman M, Sanchez C, Galve-Roperth I. Control of the cell cannabinoid receptor-dependent and -independent inhibition of survival/death decision by cannabinoids. J Mol Med 2001;78:613- depolarization-induced calcium influx in oligodendrocytes. Glia 625. 2009;57(3):295-306.

22 Neuromuscular UPDATE II

37. Anand P, Whiteside G, Fowler CJ, Hohmann AG. Targeting CB2 52. Shoemaker JL, Seely KA, Reed RL, Crow JP, Prather PL. The CB2 receptors and the endocannabinoid system for the treatment of cannabinoid agonist AM-1241 prolongs survival in a transgenic pain. Brain Res Rev 2009;60(1):255-266. mouse model of amyotrophic lateral sclerosis when initiated at 38. Maenpaa H, Mannerstrom M, Toimela T, Salminen L, Saransaari P, symptom onset. J Neurochem 2007;101(1):87-98. Tahti H. Glutamate uptake is inhibited by tamoxifen and toremifene 53. Gurney ME, Pu H, Chiu AY, Dal Canto MC, Polchow CY, Alexander in cultured retinal pigment epithelial cells. Pharmacol Toxicol DD, et al. Motor neuron degeneration in mice that express a human 2002;91(3):116-122. Cu/Zn superoxide dismutase mutation. Science 1994;264:1772- 39. Traynor BJ, Bruijn L, Conwit R, Beal F, O’Neill G, Fagan SC, 1775. Cudkowicz ME. Neuroprotective agents for clinical trials in ALS: 54. Rossi S, De Chiara V, Musella A, Cozzolino M, Bernardi G, a systematic assessment. Neurology 2006;67(1):20-27. Maccarrone M, Mercuri NB, Carri MT, Centonze D. Abnormal 40. Lee ES, Yin Z, Milatovic D, Jiang H, Aschner M. Estrogen and sensitivity of cannabinoid CB1 receptors in the striatum of mice tamoxifen protect against Mn-induced toxicity in rat cortical primary with experimental amyotrophic lateral sclerosis. Amyotroph cultures of neurons and astrocytes. Toxicol Sci 2009;110(1):156- Lateral Scler 2009;19:1-8. 167. 55. Maccarrone M, Finazzi-Agro A. The endocannabinoid system, 41. Gerra G, Zaimovic A, Gerra ML, Ciccocioppo R, Cippitelli anandamide and the regulation of mammalian cell apoptosis. Cell A, Serpelloni G, Somaini L. Pharmacology and toxicology of Death Differ 2003;10:946-955. Cannabis derivatives and endocannabinoid agonists. Recent Pat 56. Burgess RW, Cox GA, Seburn KL. Neuromuscular disease models CNS Drug Discov 2010;5(1):46-52. and analysis. Methods Mol Biol 2010;602:347-393. 42. Fischedick JT, Glas R, Hazekamp A, Verpoorte R. A qualitative 57. Bilsland LG, Dick JR, Pryce G, Petrosino S, Di Marzo V, Baker D, and quantitative HPTLC densitometry method for the analysis of Greensmith L. Increasing cannabinoid levels by pharmacological cannabinoids in Cannabis sativa L. Phytochem Anal 2009;20(5):421- and genetic manipulation delay disease progression in SOD1 mice. 426. FASEB J 2006;20(7):1003-1005. 43. Izzo AA, Borrelli F, Capasso R, Di Marzo V, Mechoulam R. Non- 58. Yiangou Y, Facer P, Durrenberger P, Chessell IP, Naylor A, Bountra psychotropic plant cannabinoids: new therapeutic opportunities C, Banati RR, Anand P. COX-2, CB2 and P2X7-immunoreactivities from an ancient herb. Trends Pharmacol Sci 2009;30(10):515- are increased in activated microglial cells/macrophages of multiple 527. sclerosis and amyotrophic lateral sclerosis spinal cord. BMC 44. Turcotte D, Le Dorze JA, Esfahani F, Frost E, Gomori A, Namaka M. Neurol 2006;6:12. Examining the roles of cannabinoids in pain and other therapeutic 59. Kapur A, Zhao P, Sharir H, Bai Y, Caron MG, Barak LS, Abood indications: a review. Expert Opin Pharmacother 2010;11(1):17- ME. Atypical responsiveness of the orphan receptor GPR55 to 31. cannabinoid ligands. J Biol Chem 2009;284(43):29817-29827. 45. Akinshola BE, Chakrabarti A, Onaivi ES. In vitro and in vivo 60. Zhao P, Leonoudakis D, Abood ME, Beattie EC. Cannabinoid action of cannabinoids. Neurochem Res 1999;24(10):1233-1240. receptor activation reduces TNF-α–induced surface localization 46. Agurell S, Halldin M, Lindgren JE, Ohlsson A, Widman M, of AMPAR-type glutamate receptors and excitotoxicity. Gillespie H, Hollister L. and metabolism of delta Neuropharmacology 2010;58(2):551-558. 1 tetrahydrocannabinol and other cannabinoids with emphasis on 61. Zhang M, Adler MW, Abood ME, Ganea D, Jallo J, Tuma RF, CB2 man. Pharmacol Rev 1986;38(1):21-43. receptor activation attenuates microcirculatory dysfunction during 47. Borrelli F, Aviello G, Romano B, Orlando P, Capasso R, Maiello cerebral ischemic/reperfusion injury. Microvasc Res 2009;78(1):86- F, Guadagno F, Petrosino S, Capasso F, Di Marzo V, Izzo AA. 94. Cannabidiol, a safe and non-psychotropic ingredient of the 62. Wallace MJ, Martin BR, DeLorenzo RJ. Evidence for a physiological marijuana plant Cannabis sativa, is protective in a murine model of role of endocannabinoids in the modulation of seizure threshold colitis. J Mol Med 2009;87(11):1111-1121. and severity. Eur J Pharmacol 2002;452:295-301. 48. Zanelati TV, Biojone C, Moreira FA, Guimarães FS, Joca SR. 63. Wallace MJ, Blair RE, Falenski KW, Martin BR, DeLorenzo RJ. Antidepressant-like effects of cannabidiol in mice: possible The endogenous cannabinoid system regulates seizure frequency involvement of 5-HT1A receptors. Br J Pharmacol 2010;159(1):122- and duration in a model of temporal lobe epilepsy. J Pharmacol 128. Exp Ther 2003;307:129-137. 49. Jones NA, Hill AJ, Smith I, Bevan SA, Williams CM, Whalley 64. Baker D, Pryce G, Giovannoni G, Thompson AJ. The therapeutic BJ, Stephens GJ. Cannabidiol displays antiepileptiform and potential of cannabis. Lancet Neurology 2003;2:291-298. antiseizure properties in vitro and in vivo. J Pharmacol Exp Ther 65. Mechoulam R, Panikashvili D, Shohami E. Cannabinoids and brain 2010;332(2):569-577. injury: therapeutic implications. Trends Mol Med 2002;8:58-61. 50. Zhao P, Leonoudakis D, Abood ME, Beattie EC. Cannabinoid 66. Richardson JD, Kilo S, Hargreaves KM. Cannabinoids reduce receptor activation reduces TNF-α–induced surface localization hyperalgesia and inflammation via interaction with peripheral CB1 of AMPAR-type glutamate receptors and excitotoxicity. receptors. Pain 1998;75:111-119. Neuropharmacology 2010;58(2):551-558. 67. Amtmann D, Weydt P, Johnson KL, Jensen MP, Carter GT. Survey 51. Yiangou Y, Facer P, Durrenberger P, Chessell IP, Naylor A, Bountra of cannabis use in patients with amyotrophic lateral sclerosis. Am J C, Banati RR, Anand P. COX-2, CB2 and P2X7-immunoreactivities Hosp Palliat Care 2004;21:95-104. are increased in activated microglial cells/macrophages of multiple 68. Carter GT, Weydt P. Cannabis: old medicine with new promise for sclerosis and amyotrophic lateral sclerosis spinal cord. BMC neurological disorders. Curr Opin Investig Drugs 2002;3(3):437- Neurol 2006;6:12. 440.

23 Alternative and Rehabilitative Approaches to Managing Neuromuscular Disease

69. Aggarwal SK, Carter GT, Sullivan MD, ZumBrunnen C, Morrill R, 87. Carter GT, Jensen MP, Stoelb BL, Hoffman AJ, Abresch RT, Mayer JD Medicinal use of cannabis in the United States: historical McDonald CM. Chronic pain in persons with myotonic muscular perspectives, current trends, and future directions. J Opioid Manag dystrophy, type 1. Arch Phys Med Rehabil 2008;89(12):2382. 2009;5(3):153-168. 88. Miro J, Raichle KA, Carter GT, O’Brien SA, Abresch RT, 70. Aggarwal SK, Carter GT, Sullivan MD, Morrill R, ZumBrunnen C, McDonald CM, Jensen MP. Impact of biopsychosocial factors on Mayer JD. Characteristics of patients with chronic pain accessing chronic pain in persons with myotonic and facioscapulohumeral treatment with medicinal cannabis in Washington State. J Opioid muscular dystrophy. Am J Hosp Palliat Med 2009;26(4):308-319. Manag 2009;5(5):257-286. 89. Engel JM, Kartin D, Carter GT, Jensen MP, Jaffe KM. Pain in 71. Carter GT, Rosen BS. Marijuana in the management of amyotrophic youths with neuromuscular disease. Am J Hosp Palliat Med lateral sclerosis. Am J Hosp Palliat Care 2001;18(4):264-270. 2009;26(5):405-412. 72. Carter GT, Jensen MP, Galer BS, Kraft GH, Crabtree LD, Beardsley 90. Bushby K, Finkel R, Birnkrant DJ, Case LE, Clemens PR, DMD Care RM, Abresch RT, Bird TD. Neuropathic Pain in Charcot Marie Considerations Working Group, et al. Diagnosis and management Tooth disease. Arch Phys Med Rehabil 1998;79:1560-1564. of Duchenne muscular dystrophy, part 2: implementation of 73. Carter GT, Butler LM, Abresch RT, Ugalde VO. Expanding the multidisciplinary care. Lancet Neurol 2010;9(2):177-189. role of hospice in the care of amyotrophic lateral sclerosis. Am J 91. Hirsch AT, Kupper AE, Carter GT, Abresch RT, McDonald Hosp and Palliat Care 1999;16(6):707-710. CM, Jensen MP. Psychosocial factors and adjustment to pain in 74. Abresch RT, Jensen MP, Carter GT. Health quality of life in peripheral individuals with postpolio syndrome. Am J Phys Med Rehabil neuropathy. Phys Med Rehabil Clin N Am 2001;12(2):461-472. 2010;89(3):213-224. 75. Gustin SM, Peck CC, Wilcox SL, Nash PG, Murray GM, 92. Lou JS, Weiss MD, Carter GT. Assessment and management Henderson LA. Different pain, different brain: thalamic anatomy of fatigue in neuromuscular disease. Am J Hosp Palliat Med in neuropathic and non-neuropathic chronic pain syndromes. J 2010;27(2):145-57. Neurosci 2011;31(16):5956-5964. 93. Carter GT, Abood ME, Aggarwal SK, Weiss MD. Cannabis 76. Abresch RT, Carter GT, Jensen MP, Kilmer DD. Assessment and amyotrophic lateral sclerosis: practical and hypothetical of pain and health-related quality of life in slowly progressive applications, and a call for clinical trials. Am J Hosp Palliat Med neuromuscular disease. Am J Hosp Palliat Care 2002;19(1):39-48. 2010;27(5):347-356. 77. Carter GT, Ugalde VO. Medical marijuana: emerging applications 94. Carter GT, Weiss MD, Chamberlain JR, Han JJ, Abresch RT, Miro J, for the management of neurological disorders. Phys Med Rehabil Jensen MP. Aging muscular dystrophy: pathophysiology and clinical Clin N Am 2004;15(4):943-954. management. Phys Med Rehabil Clin N Am 2010;21(2):429-450. 78. Jensen MP, Abresch RT, Carter GT. The reliability and validity of 95. Zeltser R, Seltzer Z, Eisen A, Feigenbaum JJ, Mechoulam R. a self-reported version of the functional independence measure in Suppression of neuropathic pain behavior in rats by a non- persons with neuromuscular disease and chronic pain. Arch Phys psychotropic synthetic cannabinoid with NMDA receptor-blocking Med Rehabil 2005;86(1):116-122. properties. Pain 1991;47(1):95-103. 79. Jensen MP, Abresch RT, Carter GT, McDonald CM. Chronic pain 96. Elikottil J, Gupta P, Gupta K. The analgesic potential of in persons with neuromuscular disorders. Arch Phys Med Rehabil cannabinoids. J Opioid Manag 2009;5(6):341-357. 2005;86(6):1155-1163. 97. Karst M, Wippermann S. Cannabinoids against pain. Efficacy and 80. Hoffman AJ, Jensen MP, Abresch RT, Carter GT. Chronic pain in strategies to reduce psychoactivity: a clinical perspective. Expert persons with neuromuscular disorders. Phys Med Rehabil Clin N Opin Investig Drugs 2009;18(2):125-133. Am 2005;16(4):1099-1112. 98. Meng ID, Manning BH, Martin WJ, Fields HL. An analgesia circuit 81. Baumrucker SJ, Carter GT, Morris GM, Stolick M, Sheldon JE, activated by cannabinoids. Nature 1998;395(6700):381-383. Brothers D. Amyotrophic lateral sclerosis and physician assisted 99. Agurell S, Halldin M, Lindgren JE, Ohlsson A, Widman M, suicide. Am J Hosp Palliat Med 2006;23(4):332-337. Gillespie H, et al. Pharmacokinetics and metabolism of delta 1 82. Carter GT, Han JJ, Abresch RT, Jensen MP. The importance of tetrahydrocannabinol and other cannabinoids with emphasis on assessing quality of life in patients with neuromuscular disease. man. Pharmacol Rev 1986;38(1):21-43. Am J Hosp Palliat Med 2007;23(6):493-497. 100. Richardson JD. Cannabinoids modulate pain by multiple 83. Jensen MP, Hoffman AJ, Stoelb BL, Abresch RT, Carter GT, mechanisms of action. J Pain 2000;1(1):1-20. McDonald CM. Chronic pain in persons with myotonic and 101. Martín-Sánchez E, Furukawa TA, Taylor J, Martin JL. Systematic facioscapulohumeral muscular dystrophy. Arch Phys Med Rehabil review and meta-analysis of cannabis treatment for chronic pain. 2008;89(2):320-328. Pain Med 2009;10(8):1353-1368. 84. Carter GT, Weiss MD, Han JJ, Chance PF, England JD. Charcot- 102. Meinck HM, Schonle PW, Conrad B. Effects of cannabinoids on Marie-Tooth Disease. Curr Treat Options Neurol 2008;10(2):94- spasticity and ataxia in multiple sclerosis. J Neurol 1989;263(2):120- 102. 122. 85. Molton I, Jensen MP, Ehde DM, Carter GT, Kraft GH, Cardenas 103. Greenberg HS, Weiness AS, Pugh JE. Short term effects of smoking D. Coping with chronic pain among younger, middle-aged, and marijuana on balance in patients with multiple sclerosis and normal older adults living with neurologic injury and disease: a role for volunteers. Clin Pharmacol Ther 1994;55:324-328. experiential wisdom. J Aging Health 2008;20:972-996. 104. Fitzgerald PB, Williams S, Daskalakis ZJ. A transcranial magnetic 86. Stoelb BL, Carter GT, Abresch RT, Purekal S, McDonald CM, stimulation study of the effects of cannabis use on motor Jensen MP. Pain in persons with postpolio synmdrome: frequency, cortical inhibition and excitability. Neuropsychopharmacology intensity, and impact. Arch Phys Med Rehabil 2008;89(10):1933- 2009;34(11):2368-2375. 1940.

24 Neuromuscular UPDATE II

105. Lichtman AH, Martin BR. Spinal and supraspinal components 123. De Lateur BJ, Giaconi RM. Effect on maximal strength of of cannabinoid-induced antinociception. J Pharmacol Exp Ther submaximal exercise in Duchenne muscular dystrophy. Am J Phys 1991;258:517-523. Med 1979;58:26-36. 106. Collin C, Davies P, Mutiboko IK, Ratcliffe S, Sativex Spasticity in 124. Eagle M. Report on the muscular dystrophy campaign workshop: MS Study Group. Randomized controlled trial of cannabis-based exercise in neuromuscular diseases Newcastle, January 2002. medicine in spasticity caused by multiple sclerosis. Eur J Neurol Neuromuscul Disord 2002;12:975-983. 2007;14(3):290-296. 125. Kilmer DD, Aitkens SG, Wright NC, McCrory MA. Response 107. Lakhan SE, Rowland M. Whole plant cannabis extracts in the to high-intensity eccentric muscle contractions in persons with treatment of spasticity in multiple sclerosis: a systematic review. myopathic disease. Muscle Nerve 2001;24:1181-1187. BMC Neurol 2010;9:59-61. 126. Kilmer DD, McCrory MA, Wright NC, Aitkens SG, Bernauer 108. Gieringer DH, Rosenthal E, Carter GT. Marijuana medical EM. The effect of a high resistance exercise program in slowly handbook: practical guide to the therapeutic uses of marijuana. progressive neuromuscular disease. Arch Phys Med Rehab Oakland, CA: Quick American Press; 2008. p. 39-109. 1994;75:560-563. 109. Abrams DI, Vizoso HP, Shade SB, Jay C, Kelly ME, Benowitz 127. Lindeman E, Leffers P, Spaans F, Drukker J, Reulen J, Kerckhoffs NL. Vaporization as a smokeless cannabis delivery system: a pilot M, Koke A. Strength training in patients with myotonic dystrophy study. Clin Pharmacol Ther 2007;82(5):572-578. and hereditary motor and sensory neuropathy: a randomized 110. Pomahacova B, Van der Kooy F, Verpoorte R. Cannabis smoke clinical trial. Arch Phys Med Rehabil 1995;76:612-620. condensate III: The cannabinoid content of vaporised Cannabis 128. Scott OM, Hyde SA, Goddard C, Jones R, Dubowitz V. Effect sativa. Inhal Toxicol 2009;21(13):1108-1112. of exercise in Duchenne muscular dystrophy. Physiotherapy 111. Carter GT, Weydt P, Kyashna-Tocha M, Abrams DI. Medicinal 1981;67:174-176. cannabis: rational guidelines for dosing. IDrugs 2004;7(5):464- 129. Wenneberg S, Gunnarsson LG, Ahlstrom G. Using a novel 470. exercise programme for patients with muscular dystrophy. Part I: a 112. Aggarwal SK, Kyashna-Tocha M, Carter GT. Dosing Medical qualitative study. Disabil Rehabil 2004;26:586-594. Marijuana: Rational Guidelines on Trial in Washington State. 130. McDonald CM, Widman LM, Walsh DD, Walsh SA, Abresch RT. MedGenMed 2007;9(3):52-53. Use of step activity monitoring for continuous physical activity 113. Breslin E, Booth J, Lord B, Carter GT, Bonekat HW, Volz B, assessment in boys with Duchenne muscular dystrophy. Arch Phys Mercer K, Siefkin A. Respiratory responses to unsupported arm Med Rehabil 2005;86:802-808. exercise (UAE) in Charcot-Marie-Tooth (CMT). Am Thorac Soc 131. Steffensen BF, Lyager S, Werge B, Rahbek J, Mattsson E. Physical Am Rev Resp Dis 1993;147:532-533. capacity in non-ambulatory people with Duchenne muscular 114. Chetlin RD, Gutmann L, Tarnopolsky M, Ullrich IH, Yeater RA. dystrophy or spinal muscular atrophy: a longitudinal study. Dev Resistance training effectiveness in patients with Charcot-Marie- Med Child Neurol 2002;44:623-632. Tooth disease: recommendations for exercise prescription. Arch 132. Lou JS, Weiss MD, Carter GT. Assessment and management Phys Med Rehabil 2004;85:1217-1223. of fatigue in neuromuscular disease. Am J Hosp Palliat Med 115. De Lateur BJ, Giaconi RM. Effect on maximal strength of 2010;27(2):145-157. submaximal exercise in Duchenne muscular dystrophy. Am J Phys 133. Krivickas LS, Taylor A, Maniar RM, Mascha E, Reisman SS. Med 1979;58:26-36. Is spectral analysis of the surface electromyographic signal a 116. Eagle M. Report on the muscular dystrophy campaign workshop: clinically useful tool for evaluation of skeletal muscle fatigue? J exercise in neuromuscular diseases Newcastle, January 2002. Clin Neurophysiol 1998;15:138-145. Neuromuscul Disord 2002;12:975-983. 134. Miro J, Raichle KA, Carter GT, O’Brien SA, Abresch RT, 117. Johnson EW, Braddom R. Over-work weakness in McDonald CM, Jensen MP. Impact of biopsychosocial factors on facioscapulohumeral muscular dystrophy. Arch Phys Med Rehabil chronic pain in persons with myotonic and facioscapulohumeral 1971;52:333-336. muscular dystrophy. Am J Hosp Palliat Med 2009;26(4):308-319. 118. Kilmer D. Response to resistive strengthening exercise training 135. McDonald CM. Limb contractures in progressive neuromuscular in humans with neuromuscular disease. Am J Phys Med Rehabil disease and the role of stretching, orthotics, and surgery. Phys Med 2002;81:S121-S126. Rehabil Clin N Am 1998;9:187-211. 119. Kilmer DD. Response to aerobic exercise training in humans with 136. Bach JR, McKeon J. Orthopedic surgery and rehabilitation for the neuromuscular disease. Am J Phys Med Rehabil 2002;81:S148- prolongation of brace- free ambulation of patients with Duchenne S150. muscular dystrophy. Am J Phys Med Rehabil 1991;70:323-331. 120. Kilmer DD. The role of exercise in neuromuscular disease. Phys 137. Bakker JP, De Groot IJ, Beelen A, Lankhorst GJ. Predictive factors Med Rehabil Clin N Am 1998;9:115-125. of cessation of ambulation in patients with Duchenne muscular 121. Topin N, Matecki S, Le Bris S, Rivier F, Echenne B, Prefaut C, dystrophy. Am J Phys Med Rehabil 2002;81:906-912. Ramonatxo M. Dose-dependent effect of individualized respiratory 138. Butler C, Okamoto G, McKay T. Motorized wheelchair driving by muscle training in children with Duchenne muscular dystrophy. disabled children. Arch Phys Med Rehab 1984;65:95-97. Neuromusc Disord 2002;12:576-583. 139. Butler C, Okamoto G, McKay T. Powered mobility for very young 122. Aitkens SG, McCrory MA, Kilmer DD, Bernauer EM. Moderate disabled children. Dev Med Child Neurol 1983;25:472-474. resistance exercise program: its effect in slowly progressive neuromuscular disease. Arch Phys Med Rehab 1993;74:711-715.

25 26 Neuromuscular Vignettes David G. Polin, MD Managing Partner The EMG Group, Ltd. Lancaster, Pennsylvania

Bakri Elsheikh, MBBS Assistant Professor of Neurology Director, EMG Laboratory The Ohio State University Medical Center Columbus, Ohio

Zachary Simmons, MD Professor of Neurology Director, Neuromuscular Program Pennsylvania State University Hershey, Pennsylvania

VIGNETTE ONE 1C. What is the MOST appropriate prescription for his upper extremity symptoms? A 55-year-old man presents with an insidious onset of painless A. Built-up eating utensils. and symmetric weakness affecting all four extremities. He is B. Dynamic hand splint. unable to get up from a low chair. He notices that his feet slap C. Myoelectric hand orthosis. when he walks and sometimes his leg will just give out suddenly D. Weight training. and he falls. You examine him and find diffuse weakness, with prominent weakness of the finger flexors (3/5) bilaterally. He VIGNETTE TWO also has quadriceps atrophy weakness of 2/5 bilaterally. Ankle dorsiflexion is 3/5 bilaterally. He has diminished reflexes. Review A 63-year-old Amish woman presents with a 2-year history of of laboratory studies reveals a serum creatine kinase (CK) level of progressive weakness and fatigue affecting the lower extremities. 484 IU/L. He has been taking simvastatin for several years. The right lower extremity is primarily affected. She has a history of polio at age 14 resulting in her being bedridden for several weeks. Questions She was given a brace but she never really liked it and threw it away. She never was able to run very well but otherwise denies 1A. What is the MOST LIKELY diagnosis? disability until about 2 years ago when her current symptoms A. Amyotrophic lateral sclerosis (ALS). began. B. Inclusion body myositis. C. Polymyositis. Examination reveals motor function to be without clear deficit in D. Toxic myopathy. the upper extremities. Motor function in the right lower extremity is 3/5 at the hip and knee and 2/5 at the ankle. Motor function 1B. What is the MOST appropriate prescription for his lower in the left lower extremity is 4/5 throughout. Sensation is intact extremity symptoms? throughout the upper and lower extremities and DTRs are diffusely A. Ankle foot orthosis (AFO) with an articulating joint. diminished. B. Posterior leaf spring (PLS) orthosis. C. Solid ankle AFO with floor reaction. Nerve conduction studies (NCSs) show CMAPs of reduced D. Long-leg brace with locking knee and articulating ankle amplitude but were otherwise unremarkable. Needle electro- joint. myography (EMG) showed very large motor unit potentials throughout the right lower extremity and in the left vastus lateralis, 27 Neuromuscular Vignettes vastus medialis, and biceps femoris. The left tibialis anterior, Questions gastrocnemius, and peroneus longus are spared. 3A. The patient MOST LIKELY has which of the following? Questions A. Acute brachial neuritis. B. ALS. 2A. What is the MOST LIKELY diagnosis? C. Cervical radiculopathy. A. ALS. D. Pancoast tumor. B. Glutaric aciduria. C. Radiculoplexus neuropathy. 3B. Which of the following would be the BEST treatment? D. Postpolio syndrome. A. Intravenous immunoglobulin (IVIg). B. PREs. 2B. Which of the following is MOST correct regarding bracing C. Steroids. for this patient? D. Watchful waiting. A. A carbon fiber low-profile AFO is indicated. B. A long-leg brace is indicated. 3C. Recovery of hand function is LIKELY to be which of the C. A myoelectric prosthosis is indicated. following? D. Do not try a brace with her because she will not accept A. A prolonged process. one. B. Minimal. C. Rapid. 2C. Regarding exercise for this patient: D. Indeterminate. A. Functional electrical stimulation has been shown to be the most effective type of exercise for this condition. 3D. Which of the following would be the BEST bracing option? B. Progressive resistive exercises (PREs) are indicated for A. Balanced forearm orthosis. only the weak muscles. B. Dynamic hand splint. C. PREs are indicated for all of the muscles. C. Thumb spica hand splint. D. Needle EMG results should be used as a guide to D. Bracing is not indicated. customize the patient’s exercise routine. VIGNETTE FOUR VIGNETTE THREE A 60-year-old woman presents with muscle stiffness and pain A 28-year-old man presents with complaint of pain and weakness since the age of 7. She recalls winters in elementary school when affecting the left upper extremity. He awoke 8 weeks ago with she was unable to unzip her coat without the teacher’s help. In severe left sided chest and shoulder pain. After a few days he addition to cold weather, her stiffness is worse with any form of noticed weakness. He was seen in the emergency room and his repeated activity or exercise. She tells you, “I learned to live with cardiac workup was unremarkable. He has since had an magnetic it, but lately I feel worse.” She reports difficulty pulling weeds resonance imaging (MRI) scan of the neck and shoulder and a from the garden or even slicing a loaf of bread. Her past medical computed tomography scan of the chest, both of which were history includes hypothyroidism and hypertension for which she unremarkable. His pain has improved over the past week and is takes levothyroxine and atenolol. She is a retired flight attendant. now down to 4/10. She has no siblings. She reports no similar history in the family, including her son and her two grandchildren. Examination reveals Examination reveals 2/5 left finger extension and 2/5 left wrist normal strength, 2+ symmetric deep tendon reflexes (DTRs), and extension. Left side intrinsics are difficult to evaluate, but there is normal pinprick and vibratory sensation. Her records indicate a resistance obtained with the fingers flat on a table. Left shoulder normal complete blood count, electrolytes, and thyroid stimulating abduction is 2/5 and left elbow extension is 3/5. Left supination hormone (TSH) and a serum CK of 75 IU/L. Needle EMG is 3/5. All other motor function tested in the upper extremities demonstrates diffuse myotonic discharges. (A video showing is at least 4+/5. Sensation is decreased to pinprick in a patchy dynamic findings on examination will be presented during the distribution in the left upper extremity and intact on the right. live course.)

NCSs reveal moderately reduced amplitude of the left radial motor Questions response with stimulating in the forearm and arm; no response is elicited with stimulating at the axilla. The left axillary motor 4A. What is the MOST LIKELY diagnosis? response was absent. The left radial sensory response also was of A. Proximal myotonic myopathy. reduced amplitude. The lateral antebrachial cutaneous response B. Dominant myotonia congenita (Thomsen’s disease). was absent on the left and easily obtainable on the right. All other C. Recessive myotonia congenita (Becker’s disease). NCSs were within normal limits. D. Paramyotonia congenita.

Needle EMG showed spontaneous activity and reduced recruitment in the biceps, extensor digitorum communis, triceps, and deltoid. Needle EMG was normal in the flexor carpi radialis, first dorsal interosseous, and abductor pollicis brevis. 28 Neuromuscular UPDATE II 4B. The responsible disorder is LIKELY associated with which 5C. Which of the following electrophysiological findings is of the following? expected in HypoPP? A. Chloride channel gene mutation. A. Postexercise myotonic potentials. B. Potassium channel gene mutation. B. Short exercise test positive for 50% initial CMAP C. Sodium channel gene mutation. decline followed by gradual recovery in 20-40 s. D. Calcium channel gene mutation. C. Long exercise test positive for 50% CMAP decline at 40 min. 4C. If you performed a short exercise test, which of the following D. During paralytic attack, CMAP amplitudes are twice the patterns would be MOST characteristic of this condition? baseline size. A. No significant change in the compound muscle action potential (CMAP). 5D. Other treatment options for HypoPP include all the following B. An initial fall in the CMAP that is less prominent with EXCEPT: repeated trials. A. Dichlorphenamide. C. An initial increase in the CMAP that is more prominent B. Trimaterene. with repeated trials. C. High-dose steroids. D. An initial fall in the CMAP that is more prominent with D. Spironolactone. repeated trials. VIGNETTE SIX 4D. This condition can be distinguished from potassium aggravated myotonia by the presence of which of the A 58-year-old female presents with 3 years of progressive following? weakness, numbness, and gait and balance difficulty. She became A. Sodium channel gene mutation. wheel chair dependent in the last year. She also reports bladder B. Extreme cold sensitivity. control difficulty and constipation. Examination reveals mild C. Myotonic potentials on needle EMG. proximal arm weakness and proximal and distal leg weakness. Her D. Age at onset of symptoms. DTRs are brisk with sustained ankle clonus and positive Hoffman’s and Babinski’s reflexes. She has decreased vibration to the knees VIGNETTE FIVE and position sense at the ankles. She has decreased pinprick sensation in a stocking and glove distribution to the midthigh and A 38-year-old man presents with attacks of weakness since the midforearm, respectively. There is no definite sensory level in age of 12. His initial symptoms were weakness of one limb or the trunk. Her previous records indicate a concern about multiple another provoked by exertion. He had an episode in high school sclerosis however MRI of the brain and cervical cord are reported where he could not stand up from his desk. He was taken to as unremarkable. Her cerebrospinal fluid analysis revealed the the hospital and was found to have low potassium of 2.5. He following: protein 27 mg/dl, glucose 75mg/dl, 0 white blood was given the diagnosis of hypokalemic periodic paralysis cells/mm3. Tests venereal disease, , and (HypoPP). His treatment regimen includes acetazolamide and oligoclonal bands were negative. potassium supplements. Despite the absence of discrete attacks of weakness, he reports that in the last couple of years he is more Other tests performed were all normal: vitamin B12 (688 pg/ml), aware of difficulty going up steps or getting up from the floor. methylmalonic acid, homocysteine, vitamin E, thyroid function test, His examination is unremarkable except for mild proximal limb rapid plasma reagin, fluorescent treponemal antibody, antinuclear weakness. antibody (ANA), extractable nuclear antigen antibodies, human immunodeficiency virus, human T-lymphotropic virus type I, and Questions lyme and neuromyelitis optica serology. A needle EMG showed an axonal sensory motor neuropathy. 5A. Which of the following is LEAST LIKELY to trigger an attack of weakness in his condition? Questions A. A long car ride. B. Rest after exercise. 6A. Review of the cervical spine MRI reveals a C3-5 posterior C. Fasting. cord signal on T2-weighted images. Which of the following D. A large carbohydrate meal. tests will LIKELY help with the diagnosis? A. Flexion and extension cervical spine MRI. 5B. HypoPP is associated with which of the following? B. Spinal cord angiography. A. Chloride channel gene mutation. C. Serum copper and zinc levels. B. Calcium channel gene mutation. D. Measurement of serum very long chain fatty acids. C. Sodium channel gene mutation. D. Both B and C. 6B. Which of the following facts is relevant to her diagnosis? A. History of L4-5 laminectomy and fusion. B. History of vegetarian diet for the last 10 years. C. History of daily use of denture cream for the last 20 years. D. History of head trauma as a child. 29 Neuromuscular Vignettes 6C. Which of the following statements is FALSE regarding the 7B. Repetitive nerve stimulation studies of the accessory nerve etiology of her condition? were performed at 3 Hz at (from left to right) pre-exercise, A. Bariatric surgery is an established cause. immediately after 30 s of isometric exercise, and 2 min B. Anemia and neutropenia may occur. postexercise. Such a finding can be found in all of the C. Vitamin B12 deficiency may coexist. following disorders EXCEPT: D. Respose to chelation.

6D. Which of the following is the MOST APPROPRIATE treatment for this patient? A. IVIg. B. High-dose oral steroids. C. Oral copper gluconate. D. Cytoxan.

VIGNETTE SEVEN A. Acquired, autoimmune myasthenia gravis. A 23-year-old woman presents for evaluation of weakness. As a B. Centronuclear myopathy. child, she could walk but not run. She eventually was given long- C. A congenital myasthenic syndrome. leg braces to wear because of leg weakness. On examination, she D. Lambert-Eaton myasthenic syndrome. had poorly developed musculature, moderate upper and lower facial weakness, moderately severe limb weakness proximally, 7C. The patient was found to have a mutation in the gene DOK-7. and well-preserved limb strength distally. Reflexes were decreased Which of the following treatments are likely to produce the or absent. Sensation was normal. She was able to arise from a BEST response? chair only with assistance, and walked with long-leg braces and A. Ephedrine and albuterol. an exaggerated lumbar lordosis. Normal or negative blood tests B. 3,4-diaminopyridine. included sedimentation rate, ANA, thyroxine (T4), TSH, vitamin C. Pyridostigmine (Mestinon®). B12, serum CK. NCSs were normal. Needle EMG revealed no D. Prednisone and IVIg. abnormal spontaneous activity. Motor unit action potentials (MUAPs) in proximal upper and lower extremity muscles were VIGNETTE EIGHT of low amplitude and short duration and demonstrated rapid recruitment. Needle examination of distal muscles was normal. A 58-year-old woman presents with a 6-month history of A muscle biopsy from the biceps muscle was interpreted as cramping in the muscles of the upper and lower extremities. She demonstrating mild myopathic changes, with increased variation also noted a stiffness in her muscles which improved with use and in fiber size and a mild increase in the number of internalized some generalized decrease in muscle strength. She had a history nuclei. of a thymoma resected 4 years earlier which was found during an evaluation for a persistent cough and was not associated with Questions any muscle weakness at that time. Examination revealed mild weakness of the neck flexors and of the distal upper extremity 7A. Which of the following is the BEST next diagnostic muscles. Reflexes were absent. Sensation was normal. She could procedure to perform in the evaluation of this patient? arise from a squatting position only by pushing with her arms. A. Biochemical testing of muscle for disorders of glycogen The ANA was 1:640. Anti-SSA and SSB antibody titers were or lipid metabolism. normal. T4 and TSH were normal. binding B. Biochemical or genetic assessment for mitochondrial antibodies were elevated to 0.17 nmol/L (normal <0.01) at the diseases. time of discovery of her thymoma, but they declined to normal C. Genetic testing for limb-girdle muscular dystrophies. after thymoma resection. Sensory and motor NCSs were normal. D. Repetitive stimulation studies of one or more nerves. Needle examination revealed abnormal spontaneous activity in the form of very high frequency (150-250 Hz) discharges in several muscles which waned in amplitude and frequency. Repetitive nerve stimulation studies of the ulnar and accessory nerves performed at 3 Hz revealed no change in the CMAP amplitude from the first to the fourth stimulus pre- or post-exercise.

Questions

8A. This disorder is LIKELY to be associated with which of the following? A. Voltage-gated potassium channel antibodies. B. Abnormally high number of CTG trinucleotide repeats. C. Voltage-gated calcium channel antibodies. D. Low levels of acid alpha glucosidase (acid maltase) 30 activity. Neuromuscular UPDATE II 8B. Which of the following is the source generator for the abnormal discharges seen on the needle EMG examination? A. Muscle fiber. B. Neuromuscular junction. C. Motor unit (motor neuron/axon). D. Cortical motor neuron.

8C. Appropriate treatments for this condition include each of the following EXCEPT: A. Carbamazepine. B. Amitriptyline. C. Prednisone. D. Phenytoin.

VIGNETTE NINE

A 51-year-old man presents with a 2-year history of lower extremity weakness, beginning with difficulty climbing stairs. This persisted and worsened. His maternal grandfather and the grandfather’s brother both experienced progressive leg weakness as they became older, without a specific diagnosis. Examination revealed mild tongue atrophy, with tongue and chin fasciculations. There was mild weakness of the spinati and of the hip flexors. Reflexes were absent. There was mild distal sensory loss in the feet. Gait was normal, but he arose from a squatting position by pushing with his arms. Laboratory studies were remarkable for serum CK levels of 1,259 and 1,372 IU/L. Normal or negative studies included sedimentation rate, ANA, T4, and TSH. NCSs are shown below.

Sensory Amplitude Peak latency Distance Conduction velocity nerve (all left) (µV) (ms) (mm) (m/s)

Median 8.8 3.3 140 54.2 Ulnar 6.6 3.4 140 50.0 Sural 4.2 3.2 140 53.8

Motor nerve Amplitude Onset Distance Conduction (all left) (mV) latency (mm) velocity (ms) (m/s)

Median Wrist 6.39 4.6 70 Elbow 5.20 8.7 220 53.6 Ulnar Wrist 8.45 3.1 70 Below elbow 7.17 7.2 210 51.2 Above elbow 6.93 8.7 100 66.6 Peroneal Ankle 2.43 4.9 90 Fibular head 1.80 12.3 320 43.2 Popliteal fossa 1.69 13.8 80 53.3 Tibial Ankle 5.95 6.8 80 Popliteal fossa 5.49 14.5 425 55.1

Needle examination revealed fibrillation potentials and positive sharp waves in muscles of the right upper and lower extremities. MUAPs in virtually all muscles tested, proximally and distally, were of large size and demonstrated decreased recruitment.

31 Neuromuscular Vignettes Questions Questions

9A. This patient is LIKELY to have which of the following 10A. The predominant finding in a muscle biopsy from this patient conditions? is MOST LIKELY to be which of the following? A. Cardiomyopathy. A. B. Retinal pigmentary degeneration. C. Gynecomastia. D. Cataracts.

9B. Inheritance of this disease is which of the following? A. Autosomal recessive. B. Via mitochondrial DNA. B. C. Autosomal dominant with incomplete penetrance. D. X-linked.

9C. The underlying genetic basis for this condition is which of the following? A. CTC trinucleotide repeat expansion on the DMPK gene. B. CAG trinucleotide repeat expansion on the androgen C. receptor gene. C. CAG trinucleotide repeat expansion on the HTT (Huntington’s disease) gene. D. CCTG repeat expansion on the CNBP (zinc finger protein 9) gene.

VIGNETTE TEN D. A 17-year-old girl presents for evaluation of a 4-year history of progressive, proximal muscle weakness, characterized by difficulty climbing stairs even when pulling on the railing, and a need to push with her arms when arising from a chair. Her leg weakness had resulted in several falls. Family history was negative for any disorder producing muscle weakness. Examination revealed a very thin young woman with normal mental status and normal cranial nerve examinations. She demonstrated full strength in the upper extremities. Lower extremities were characterized by moderately severe hip flexor weakness and mild-to-moderate 10B. A muscle biopsy revealed a vacuolar myopathy with increased distal weakness. Reflexes were absent. Sensation was normal. She variation in fiber size and an increased number of internalized arose from a chair by pushing with her arms on her knees. Gait was nuclei. Areas of positive staining with periodic acid-Schiff characterized by an exaggerated lumbar lordosis and a swaying of were noted. Electron microscopy demonstrated glycogen the hips. Laboratory studies demonstrated a serum CK level of both within the vacuoles and freely in the cytoplasm. The 1,158 IU/L. Sedimentation rate and thyroid function tests were etiology of this patient’s condition MOST LIKELY is due to normal. Sensory and motor NCSs of an upper and lower extremity an abnormality in which of the following? were normal. Needle EMG examination revealed fibrillation A. The DMPK gene. potentials, positive sharp waves, and myotonic discharges in B. The GNE gene. several muscles, more prominent in the lower than the upper C. Mitochondrial DNA. extremities, more prominent proximally than distally. MUAPs D. The GAA gene. were of short duration, small amplitude, and demonstrated early recruitment. 10C. Biochemical analysis of the muscle revealed a markedly reduced level of acid maltase activity of 0.38 mcmol/min/g tissue (normal 1.74-9.98). In view of this finding, which of the following is MOST LIKELY to occur? A. Cardiomyopathy and heart failure. B. Hepatomegaly and liver failure. C. Respiratory muscle weakness and respiratory failure. D. Retinal degeneration and blindness.

32 Neuromuscular UPDATE II

ICU Related Neuromuscular Complications, Alternative and Rehab Approaches to Managing NMD, and Neuromuscular Vignettes (Neuromuscular Update II) CME Questions

1. A combination of steroids and neuromuscular blocking 6. A 27-year-old male presents with numbness and weakness agents in the intensive care unit has been associated with: in the lower extremities. He has a family history of Charcot- A. Nemaline rod myopathy. Marie-Tooth disease. On examination, he demonstrates B. Cachectic (disuse) myopathy. significant 4/5 strength in most major muscle groups, except C. Polymyositis. dorsiflexion where he is 3/5 bilaterally. After walking for D. Critical illness myopathy. several minutes, he does start to drag his feet. Which type E. Critical illness neuropathy. of bracing modality is most likely to improve his gait? A. Custom fit shoe orthotics with metatarsal pads. 2. A characteristic muscle biopsy finding in critical illness B. Bilateral universal foot orthotics. myopathy is: C. Bilateral long leg double metal upright knee ankle-foot A. Inflammatory cellular infiltrate. orthotics. B. Extensive muscle fibers necrosis. D. Bilateral double metal upright ankle-foot orthotics C. Thick filaments (myosin) loss. (AFOs). D. Grouped muscle fibers atrophy. E. Light weight, low profile, AFOs with dorsiflexion E. Variation in muscle fibers size and shape. assist.

3. Which of the following electrophysiological findings is NOT 7. Which type of exercise is most likely to damage muscle and associated with critical illness myopathy? potentially contribute to loss of strength in a patient with A. Low compound muscle action potential (CMAP) limb girdle muscular dystrophy? amplitude. A. Progressive endurance training in a pool. B. Short duration, small amplitude polyphasic motor unit B. Submaximal weight training in muscles with greater potential. than 3/5 strength on manual muscle testing. C. Irritative changes (fibrillations and positive waves). C. Eccentric resistance training to failure in muscles with D. Occasional mild sensory abnormalities. less than 3/5 strength on manual muscle testing. E. CMAP amplitude changes on repetitive nerve D. Maximal isometric contractions in pelvic girdle stimulation. musculature. E. None of the above. 4. Which of the following features is most characteristic of prolonged neuromuscular junction block? 8. Which gait characteristic is LEAST likely to be helped by A. Flaccid weakness of all limbs (tetraplegia). orthotics in a patient with Becker muscular dystrophy? B. Depressed or absent tendon reflexes (areflexia). A. Toe walking. C. Facial and extraocular muscles weakness. B. Pes cavus with equino varus. D. Sepsis and multi-organ failure. C. Gluteus medius gait. E. Failure to wean from mechanical ventilator. D. Foot drop with a steppage gait. E. None of the above. 5. Risk factors for acquired neuromuscular weakness in the ICU include all the following EXCEPT: A. High doses of pancuronium or vecuronium. B. High doses of IV corticosteroids. C. Systemic inflammatory response syndrome. D. Known history of asthma or organ transplant. E. Known history of diabetic polyneuropathy.

33 CME QUESTIONS 9. Regarding the use of medicinal cannabis in managing 10. Cannabinoids, the active ingredients in medicinal cannabis, amyotrophic lateral sclerosis (ALS), which statement is the have been shown to have all of the following properties most appropriate and correct? EXCEPT? A. Cannabis would not be helpful for a patient with ALS. A. Neuroprotection. B. ALS patients have self-reported that cannabis is helpful B. Anti-inflammatory, both centrally and peripherally. to control pain, muscle spasms, drooling, loss of C. Retrograde transmission in the central nervous system. appetite, and diminished mood state. D. Strong binding to respiratory centers in the brainstem. C. Pre-clinical studies have shown no effect of E. Analgesia via a mechanism that is synergistic but cannabinoids in a mouse model of ALS (SOD1 mouse). distinct from opiates. D. ALS patients cannot use cannabis because smoking is contraindicated. E. There is no way any ALS patient in the United States can use medical cannabis because it is classified as a schedule I drug by the Drug Enforcement Agency.

34 VISIT THE AANEM MARKETPLACE AT WWW.AANEM.ORG FOR NEW PRODUCTS

American Association of Neuromuscular & Electrodiagnostic Medicine 2621 Superior Dr NW Rochester, MN 55901 T: 507.288.0100 F: 507.288.1225 www.aanem.org